Tag Archives: 2017

ĐỀ THI CHỌN ĐỘI TUYỂN HSG QUỐC GIA CỦA TRƯỜNG PTNK NĂM 2016 – 2017

ĐỀ THI

Ngày thi thứ nhất

Bài 1. Tìm tất cả $a$ để dãy số $\left(u_n\right)$ hội tụ, biết $u_1=a$ và $\forall n \in \mathbb{N}^*$ thì:

$\quad\quad\quad\quad\quad\quad\quad\quad\quad\quad u_{n+1}=\left\{\begin{array}{l}2 u_n-1 \text { nếu } u_n>0 \\ -1 \text { nếu }-1 \leq u_n \leq 0 \\ u_n^2+4 u_n+2 \text { nếu } u_n<-1\end{array}\right.$

Bài 2. Tìm số nguyên dương $k$ nhỏ nhất để bất đẳng thức

$\quad\quad\quad\quad\quad\quad\quad\quad\quad\quad x^k y^k z^k\left(x^3+y^3+z^3\right) \leq 3$

luôn đúng với mọi số thực dương $x, y, z$ thoả mãn điều kiện $x+y+z=3$.

Bài 3. Cho hàm số $f: \mathbb{N}^* \rightarrow \mathbb{N}^*$ thoả mãn hai điều kiện sau:

$\quad\quad$ i) $f$ là hàm tăng thật sự trên $\mathbb{N}^*$.

$\quad\quad$ ii) $f(2 n)=2 f(n) \forall n \in \mathbb{N}^*$.

(a) Giả sử $f(1)=3$ và $p>3$ là số nguyên tố. Chứng minh rằng tồn tại số nguyên dương $n$ sao cho $f(n)$ chia hết cho $p$.

(b) Cho $q$ là số nguyên tố lẻ. Hãy xây dựng một hàm $f$ thoả mãn các điều kiện của bài toán mà $f(n)$ không chia hết cho $q$ với mọi $n$ nguyên dương.

Bài 4. Cho tam giác $A B C$ có góc $\angle B A C$ tù và $A H \perp B C(H$ nằm trên $B C)$. Điểm $M$ thay đổi trên cạnh $A B$. Dựng điểm $N$ sao cho $\Delta B M N \sim \triangle H C A$, với $H$ và $N$ nằm khác phía đối với đường thẳng $A B$.

(a) Gọi $C M$ cắt đường tròn ngoại tiếp tam giác $B M N$ tại $K$. Chứng minh rằng $N K$ luôn đi qua một điểm cố định.

(b) Gọi $N H$ cắt $A C$ tại $P$. Dựng điểm $Q$ sao cho $\triangle H P Q \sim \triangle H N M$, với $Q$ và $M$ nằm khác phía đối với đường thẳng $N P$. Chứng minh rằng $Q$ luôn thuộc một đường thẳng cố định.

Ngày thi thứ hai

Bài 5. Với mỗi số nguyên dương $n$, tồn tại duy nhất số tự nhiên $a$ thoả mãn điều kiện $a^2 \leq n<(a+1)^2$. Đặt $\Delta_n=n-a^2$.

(a) Tìm giá trị nhỏ nhất của $\Delta_n$ khi $n$ thay đổi và luôn thoả mãn $n=15 m^2$ với $m$ là số nguyên dương.

(b) Cho $p, q$ là các số nguyên dương và $d=5(4 p+3) q^2$. Chứng minh rằng $\Delta_d \geq 5$.

Bài 6. Với các số nguyên $a, b, c, d$ thoả mãn $1 \leq a<b<c<d$, ký hiệu:

$T(a, b, c, d)=[(x, y, z, t) \subset \mathbb{N}^* \mid 1 \leq x<y<z<t, x \leq a, y \leq b, z \leq c, t \leq d]$

(a) Tình số phần tử của $T(1,4,6,7)$.

(b) Cho $a=1$ và $b \geq 4$. Gọi $d_1$ là số phần tử của $T(a, b, c, d)$ chứa 1 và không chứa $2 ; d_2$ là số phần tử chứa 1,2 và không chứa $3 ; d_3$ là số phần tử chứa $1,2,3$ và không chứa 4 . Chứng minh rằng $d_1 \geq 2 d_2-d_3$. Đẳng thức xảy ra khi nào?

Bài 7. Trong một hệ thống máy tính, một máy tính bất kỳ có kết nối trực tiếp với ít nhất $30 \%$ máy tính khác của hệ thống. Hệ thống này có một chương trình cảnh báo và ngăn chặn khá tốt, do đó khi một máy tính bị virus, nó chỉ có đủ thời gian lây cho các máy tính được kết nối trực tiếp với nó. Chứng minh rằng dù vậy, kẻ tấn công vẫn có thể chọn hai máy tính của hệ thống mà nếu thả virus vào hai máy đó, ít nhất $50 \%$ máy tính của hệ thống sẽ bị nhiễm virus.

Bài 8. Cho tam giác $A B C$ nhọn. Đường tròn $(I)$ có tâm $I$ thuộc cạnh $B C$ và tiếp xúc với các cạnh $A B, A C$ lần lượt tại $E, F$. Lấy $M, N$ bên trong tứ giác $B C E F$ sao cho $E F N M$ nội tiếp $(I)$ và các đường thẳng $M N, E F, B C$ dồng quy. Gọi $M F$ cắt $N E$ tại $P, A P$ cắt $B C$ tại $D$.

(a) Chứng minh rằng $A, D, E, F$ cùng thuộc một đường tròn.

(b) Lấy trên các đường thẳng $B N, C M$ các điểm $H, K$ sao cho $\angle A C H=$ $\angle A B K=90^{\circ}$. Gọi $T$ là trung điểm $H K$. Chứng minh rằng $T B=T C$.

 

LỜI GIẢI

Ngày thi thứ nhất

Bài 1. Tìm tất cả $a$ để dãy số $\left(u_n\right)$ hội tụ, biết $u_1=a$ và $\forall n \in \mathbb{N}^*$ thì:

$\quad\quad\quad\quad\quad\quad\quad\quad\quad\quad u_{n+1}=\left\{\begin{array}{l}2 u_n-1 \text { nếu } u_n>0, \\ -1 \text { nếu }-1 \leq u_n \leq 0, \\ u_n^2+4 u_n+2 \text { nếu } u_n<-1\end{array}\right.$

Lời giải. Có các trường hợp sau cần xem xét:

  • Nếu $a>1$, bằng quy nạp đơn giản, ta có $u_n>1 \forall n \in \mathbb{N}^*$ và

$\quad\quad\quad\quad\quad\quad\quad\quad\quad\quad u_n=2^{n-1}(a-1)+1, \forall n \in \mathbb{N}^* .$

Do $a>1$, cho $n \rightarrow+\infty$ thì $u_n \rightarrow+\infty$. Từ đó $\left(u_n\right)$ không hội tụ.

  • Nếu $a=1$ thì $u_n=1 \forall n \in \mathbb{N}^*$ hay $\left(u_n\right)$ hội tụ về 1 .

  • Nếu $0<a<1$, ta sẽ chứng minh rằng $\left(u_n\right)$ có ít nhất một số hạng không dương. Thật vậy, giả sử $u_n>0 \forall n \in \mathbb{N}^*$ thì theo trường hợp đầu tiên, ta có:

$\quad\quad\quad\quad\quad\quad\quad\quad\quad\quad u_n=2^{n-1}(a-1)+1 \forall n \in \mathbb{N}^*$

Do $a>1$, cho $n \rightarrow+\infty$ thì $u_n \rightarrow-\infty$, trái với việc $u_n>0 \forall n, \in \mathbb{N}^*$.

Từ đó điều giả sử là sai hay phải tồn tại $k \in \mathbb{N}^*\text { sao cho } u_k>0 \text { và } u_{k+1} \leq 0$. Với cách chọn chỉ số $\text{k}$ như vậy, ta có:

$\quad\quad\quad\quad\quad\quad\quad\quad\quad\quad -1 \leq 2 u_k-1=u_{k+1} \leq 0$

Khi đó $u_{k+2}=0$. Bằng quy nạp thì $u_n=-1 \forall n \in \mathbb{N}^*, n \geq k+2$. Điều này dễn đến $\left(u_n\right)$ hội tụ về $-1$.

  • Nếu $-1 \leq a \leq 0$, từ giả thiết thì $u_2=-1$. Bằng quy nạp thì $u_n=-1 \forall n \in$ $\mathbb{N}^*, n \geq 2$ hay $\left(u_n\right)$ hội tụ về $-1$.

  • Nếu $-2<a<-1$, ta có:

$\quad\quad\quad\quad\quad\quad\quad\quad\quad\quad u_2-u_1=a^2+3 a+2=(a+2)(a+1)<0$

Khi đó thì $u_2<u_1<-1$. Lại có $u_2=(a+2)^2-2 \geq-2$ nên $-2<u_2<-1$.

Bằng quy nạp, ta có $\left(u_n\right)$ là dãy giảm và $-2<u_n<-1$ nên $\left(u_n\right)$ hội tụ.

  • Nếu $-2-\sqrt{3} \leq a \leq-2$ thì $u_2=a^2-4 a+2$ và dễ có được:

$\quad\quad\quad\quad\quad\quad\quad\quad\quad\quad\quad\quad -1 \leq a^2-4 a+2 \leq 1$

Theo các trường hợp đã xét, dãy số $\left(u_n\right)$ hội tụ.

  • Nếu $a<-2-\sqrt{3}$, bằng vài tính toán, ta có $u^2=a^2-4 a+2>1$.

Theo trường hợp đầu tiên, dãy số $\left(u_n\right)$ không hội tụ.

Vậy dãy số $\left(u_n\right)$ hội tụ khi và chỉ khi $-2-\sqrt{3} \leq a \leq 1$.

Bài 2. Tìm số nguyên dương $k$ nhỏ nhất để bất đẳng thức

$\quad\quad\quad\quad\quad\quad\quad\quad\quad\quad x^k y^k z^k\left(x^3+y^3+z^3\right) \leq 3$

luôn đúng với mọi số thực dương $x, y, z$ thoả mãn điều kiện $x+y+z=3$.

Lời giải. Ta sẽ chứng minh rằng $k=3$ là số nguyên dương nhỏ nhất thoả mãn bài toán. Trước hết, chọn $x=y=\frac{3}{4}, z=\frac{3}{2}$ thì ta phải có:

$\quad\quad\quad\quad\quad\quad\quad\quad \left(\frac{3}{4}\right)^{2 k} \cdot\left(\frac{3}{2}\right)^k\left(2 \cdot\left(\frac{3}{4}\right)^3+\left(\frac{3}{2}\right)^3\right) \leq 3$

Dễ thấy đánh giá trên chỉ đúng nếu $k \geq 3$. Ta đưa về chứng minh rằng:

$\quad\quad\quad\quad\quad\quad\quad\quad\quad\quad x^3 y^3 z^3\left(x^3+y^3+z^3\right) \leq 3 .$

Không mất tính tổng quát, giả sử $x \geq y \geq z$ thì $z \leq 1$. Ta có:

$\quad\quad\quad\quad\quad x^3+y^3=(x+y)^3-3 x y(x+y)=(3-z)^3-3 x y(x+y) \text { hay } $

$\quad\quad\quad\quad\quad (3-z)^3+z^3 \leq \frac{3}{x^3 y^3 z^3}+3 x y(x+y)$

Khai triển và thu gọn, bất đẳng thức trở thành:

$\quad\quad\quad\quad\quad\quad\quad\quad\quad\quad 3 z^2-9 z+9 \leq \frac{1}{x^3 y^3 z^3}+x^2 y+x y^2$

Theo bất đẳng thức AM-GM, ta có vế phải của bất đẳng thức trên sẽ không nhỏ hơn $\frac{3}{z}$. Từ đây ta chỉ cần chứng minh rằng

$\quad\quad\quad\quad\quad\quad\quad 3 z^2-9 z+9 \leq \frac{3}{z} \text { hay } 3(z-1)^3 \leq 0 \text {, đúng. }$

Vậy $k=3$ là hằng số nguyên dương nhỏ nhất thoả mãn bài toán.

Nhận xét. Dưới đây là các cách xử lý khác cho bất đẳng thức ứng với $k=3$ ở trên.

Cách 1. Không mất tính tổng quát ta giả sử $x \leq y \leq z$. Khi đó luôn tồn tại $m>n \geq 0$ sao cho $x=m-n, y=m+n$. Khi đó

$\quad\quad\quad\quad\quad\quad\quad\quad\quad\quad z=3-2 m ; m=\frac{x+y}{2} \leq 1$

Xét hàm số

$f(n)=(m-n)^3(m+n)^3 z^3\left[z^3+(m-n)^3+(m+n)^3\right]=z^3\left(m^2-n^2\right)^3\left(z^3+2 m^3+6 m n^2\right)$

thì

$\quad\quad\quad\quad\quad f^{\prime}(n)=z^3\left(m^2-n^2\right)^2\left(-6 n z^3-48 m n^3\right) \leq 0$

nên

$\quad\quad\quad\quad\quad f(n) \leq f(0)=m^6 z^3\left(z^3+2 m^3\right)=m^6(3-2 m)^3\left((3-2 m)^3+2 m^3\right)$

Xét hàm số

$\quad\quad\quad\quad\quad\quad\quad\quad\quad\quad g(m)=m^6(3-2 m)^3\left[(3-2 m)^3+2 m^3\right]$

thì

$\quad\quad\quad\quad\quad g^{\prime}(m)=18 m^5(3-2 m)^2(m-1)\left[(m-1)\left(8 m^2-37 m+26\right)-1\right] \geq 0 .$

Vậy nên $g(m) \leq g(1)=3$, bài toán được giải quyết.

Cách 2. Không mất tính tổng quát, ta giả sử $z$ là số lớn nhất trong ba số $x, y, z$. Đặt $t=\frac{x+y}{2}$ và $f(x, y, z)=x^3 y^3 z^3\left(x^3+y^3+z^3\right)$. Ta sẽ chứng minh $f(x, y, z) \leq$ $f(t, t, z)$. Ta có

$\quad\quad\quad\quad f(t, t, z)-f(x, y, z)=z^3\left[t^6\left(2 t^3+z^3\right)-x^3 y^3\left(x^3+y^3+z^3\right)\right] .$

Mặt khác,

$t^6\left(2 t^3+z^3\right)-x^3 y^3\left(x^3+y^3+z^3\right)=z^3\left(t^6-x^3 y^3\right)+2 t^9-x^3 y^3(x+y)\left(x^2+y^2-x y\right) $

$=z^3\left(t^6-x^3 y^3\right)+2 t^9-2 t x^3 y^3\left(4 t^2-3 x y\right) \geq t^3\left(t^6-x^3 y^3\right)+2 t^9-2 t x^3 y^3\left(4 t^2-3 x y\right) $

$=3 t\left(t^2-x y\right)\left[t^6+x y\left(2 x y+t^2\right)\left(t^2-x y\right)\right] \geq 0 .$

Vậy nên

$\quad\quad\quad\quad\quad f(x, y, z) \leq f(t, t, z)=f(t, t, 3-2 t)=t^6(3-2 t)^3\left[2 t^3+(3-2 t)^3\right]$

Ta chỉ cần chứng minh

$\quad\quad\quad\quad\quad\quad\quad\quad\quad\quad t^6(3-2 t)^3\left[2 t^3+(3-2 t)^3\right] \leq 3$

Đến đây thực hiện như cách 1 ở trên.

Bài 3. Cho hàm số $f: \mathbb{N}^* \rightarrow \mathbb{N}^*$ thoả mãn hai điều kiện sau:

i) $f$ là hàm tăng thật sự trên $\mathbb{N}^*$.

ii) $f(2 n)=2 f(n) \forall n \in \mathbb{N}^*$.

(a) Giả sử $f(1)=3$ và $p>3$ là số nguyên tố. Chứng minh rằng tồn tại số nguyên dương $n$ sao cho $f(n)$ chia hết cho $p$.

(b) Cho $q$ là số nguyên tố lẻ. Hãy xây dựng một hàm $f$ thoả mãn các điều kiện của bài toán mà $f(n)$ không chia hết cho $q$ với mọi $n$ nguyên dương.

Lời giải. (a) Đặt $A=[f(n+1)-f(n) \mid n \in \mathbb{N}^*].$

Vì $\text { f là hàm số tăng thực sự trên } \mathbb{N}^* \text { nên } A \subset \mathbb{N}^*$.

Khi đó phải tồn tại $k=\min A \text { và tồn tại } n \in \mathbb{N}^* \text { để } k=f(n+1)-f(n)$. Khi đó:

$\quad\quad\quad\quad\quad\quad\quad\quad\quad\quad f(2 n+2)-f(2 n)=2 f(n+1)-2 f(n)=2 k .$

Lại có $f(2 n+2)-f(2 n+1), f(2 n+1)-f(2 n) \geq k$ nên

$\quad\quad\quad\quad\quad\quad\quad\quad\quad\quad f(2 n+2)-f(2 n+1)+f(2 n+1)-f(2 n) \geq 2 k .$

Từ đây ta phải có $f(2 n+2)-f(2 n+1)=f(2 n+1)-f(2 n)=k$. Bằng quy nạp theo $m$, ta chứng minh được

$\quad\quad\quad\quad\quad\quad\quad\quad\quad\quad f\left(2^m n+t\right)=2^m f(n)+t k \forall t, m \in \mathbb{N}, t \leq m .$

Lại có $f(1)=3, f(2)=6$ nên $k \leq 3<p$ hay $(k, p)=1$.

Xét $p$ số nguyên dương sau:

$\quad\quad\quad\quad\quad\quad\quad\quad\quad\quad f\left(2^p n\right), f\left(2^p n+1\right), f\left(2^p n+2\right), \ldots, f\left(2^p n+p-1\right)$

lập thành một cấp số cộng có công sai $k$ nên là một hệ thặng dư đầy đủ modulo $p$. Từ đó phải tồn tại một số hạng chia hết cho $p$.

(b) Ta xây dựng một hàm số $f$ với các điều kiện như sau:

$\quad\quad$ i) $f(1)=2^a>q\left(a \in \mathbb{N}^*\right.$,

$\quad\quad$ ii) $f(2 n)=2 f(n) \forall n \in \mathbb{N}^*$,

$\quad\quad$ iii) $f(2 n+1)=f(2 n)+q \forall n \in \mathbb{N}^*$.

Ta chứng minh rằng hàm số $f$ vừa xây dựng thỏa mãn bài toán.

Trước hết ta chứng minh rằng $f$ là hàm tăng thực sự, cụ thể là:

$\quad\quad\quad\quad\quad\quad\quad\quad\quad\quad f(n+1)-f(n) \geq q \forall n \in \mathbb{N}^* .$

Với $n=1$, ta có $f(2)-f(1)=2.2^a-2^a=2^a>q$. Giả sử khẳng định cần chứng minh đúng đến $n=k$. Xét các khả năng sau:

  • Nếu $k$ là số chẵn, ta có $f(k+1)=f(k)+q$ thỏa mãn yêu cầu.

  • Nếu $k$ là số lẻ, ta có:

$\quad\quad\quad\quad f(k+1)=2 f\left(\frac{k+1}{2}\right) \geq 2\left(f\left(\frac{k-1}{2}\right)+q\right)=f(k-1)+2 q .$

Lại có $f(k)=f(k-1)+q$ nên $f(k+1) \geq f(k)+q$.

Theo nguyên lý quy nạp, ta có $f(n+1)-f(n) \geq q \forall n \in \mathbb{N}^*$.

Bây giờ ta chứng minh rằng không tồn tại $n$ để $q \mid f(n)$. Trước hết thì $f(1)=2^a$ không chia hết cho $q$. Giả sử điều này đúng đến $n=k$. Xét các khả năng sau:

  • Nếu $k$ chẵn thì $f(k+1)=f(k)+q$ không chia hết cho $q$.

  • Nếu $k$ lẻ thì $f(k+1)=2 f\left(\frac{k+1}{2}\right)$ không chia hết cho $q$.

Theo nguyên lý quy nạp, $f(n)$ không chia hết cho $q$ với mọi $n \in \mathbb{N}^*$. Các điều kiện đã được kiểm tra đầy đủ.

Bài 4. Cho tam giác $A B C$ có góc $\angle B A C$ tù và $A H \perp B C(H$ nằm trên $B C$ ). Điểm $M$ thay đổi trên cạnh $A B$. Dựng điểm $N$ sao cho $\Delta B M N \sim \triangle H C A$, với $H$ và $N$ nằm khác phía đối với đường thẳng $A B$.

(a) Gọi $C M$ cắt đường tròn ngoại tiếp tam giác $B M N$ tại $K$. Chứng minh rằng $N K$ luôn đi qua một điểm cố định.

(b) Gọi $N H$ cắt $A C$ tại $P$. Dựng điểm $Q$ sao cho $\triangle H P Q \sim \Delta H N M$, với $Q$ và $M$ nằm khác phía đối với đường thẳng $N P$. Chứng minh rằng $Q$ luôn thuộc một đường thẳng cố định.

Lời giải. (a) Xét điểm $X$ trên $A C$ sao cho $\angle X B C=90^{\circ}$ và $K^{\prime}$ là giao điểm của $N X$ và $C M$. Ta có $\Delta B M N \sim \triangle B C X$ (cùng hướng). Từ đó có một phép vị tự quay tâm $B$ biến $M \mapsto N, C \mapsto X$.

Giả sử $C M$ cắt $B X$ tại $K^{\prime}$ thì $K^{\prime}$ thuộc đường tròn ngoại tiếp tam giác $B M N$. Từ đó $K^{\prime} \equiv K$ nên $N K$ luôn đi qua điểm $X$ cố định.

(b) Xét phép vị tự tâm $H$ biến

$\quad\quad\quad\quad\quad\quad\quad\quad\quad\quad N \mapsto P, M \mapsto Q, B \mapsto F .$

Ta có $\Delta B M N \sim \triangle F Q P$. Khi đó

$\quad\quad\quad\quad\quad\quad\quad\quad\quad\quad \angle F Q P=\angle B M N=\angle A C B=\angle F C P$

nên tứ giác $C F P Q$ nội tiếp. Từ đây dẫn đến

$\quad\quad\quad\quad\quad\quad\quad\quad\quad\quad \angle Q C P=\angle Q F P=\angle M B N=90^{\circ} .$

Vậy $Q$ thuộc đường thẳng qua $C$ vuông góc với $A C$, là đường thẳng cố định.

 

Ngày thi thứ hai

Bài 5. Với mỗi số nguyên dương $n$, tồn tại duy nhất số tự nhiên $a$ thoả mãn điều kiện $a^2 \leq n<(a+1)^2$. Đặt $\Delta_n=n-a^2$.

(a) Tìm giá trị nhỏ nhất của $\Delta_n$ khi $n$ thay đổi và luôn thoả mãn $n=15 m^2$ với $m$ là số nguyên dương.

(b) Cho $p, q$ là các số nguyên dương và $d=5(4 p+3) q^2$. Chứng minh rằng $\Delta_d \geq 5$.

Lời giải. (a) Ta cần tìm $\Delta_n$ nhỏ nhất để phương trình $15 m^2-a^2=\Delta_n$ có nghiệm nguyên dương. Nhận thấy $15-3^2=6$ nên $\min \Delta_n \leq 6$. Ta chứng minh rằng phương trình trên không có nghiệm nguyên dương với $\Delta_n<6$.

Ta có $3 \mid a^2+\Delta_n$. Suy ra $3 \mid \Delta_n$ hoặc $3 \mid \Delta_n+1$. Mặt khác $5 \mid a^2+\Delta_n$ nên $\Delta_n$ chia 5 chỉ có thể dư 0,1 hoặc 4 .

Từ đó nếu tồn tại $n$ để $\Delta_n<6$ thỏa mãn bài toán thì $\Delta_n=5$. Giả sử rằng tồn tại $n$ như thế, ta có $15 m^2-a^2=5$ hay $5 \mid a$. Đặt $a=5 s\left(s \in \mathbb{N}^*\right)$, ta có:

$\quad\quad\quad\quad\quad\quad\quad\quad\quad\quad\quad\quad 3 m^2-5 s^2=1 \text {. }$

Từ đó thì

$\quad\quad\quad\quad 3\left(m^2+s^2\right) \equiv 1 \quad(\bmod 8)$ hay $m^2+s^2 \equiv 3 \quad(\bmod 8)$

Điều này vô lý do $m^2$ chia 8 dư $0,1,4$. Vậy $\Delta_n$ nhỏ nhất là 6 .

(b) Ta có

$\quad\quad\quad\quad\quad\quad\quad\quad\quad\quad 5(4 p+3) q^2-a^2=\Delta_d .$

Do $a^2$ chia 5 dư $0,1,4$ nên $\Delta_d$ chia 5 dư $0,1,4$. Giả sử rằng có bộ số để $\Delta_d<5$. Xét các khả năng sau:

  • Nếu $\Delta_d=0$ thì $5(4 p+3) q^2=a^2$. Xét bộ số $(q, a)$ với $q+a$ nhỏ nhất. Từ phương trình trên, ta có $a^2+q^2 \equiv 0(\bmod 4)$ hay $a \equiv q \equiv 0(\bmod 2)$.

Đặt $a=2 a_1$ và $q=2 q_1$ với $a_1, q_1 \in \mathbb{N}^*$ thì bộ số $\left(q_1, a_1\right)$ cũng thoả mãn điều kiện $5(4 p+3) q_1^2=a_1^2$. Hơn nữa $q_1+a_1<q+a$, mâu thuẫn.

  • Nếu $\Delta_d=1$, ta có $a^2+1=5(4 p+3) q^2$. Do $5(4 p+3) \equiv 3(\bmod 4)$ nên số này tồn tại một ước nguyên tố $r \equiv 3(\bmod 4)$.

Do đó $a^2+1 \equiv 0(\bmod r)$ hay $r \mid 1$, vô lý.

  • Nếu $\Delta_d=4$, chứng minh tương tự, ta cũng có điều mâu thuẫn.

Vậy ta phải có $\Delta_d \geq 5$.

Bài 6. Với các số nguyên $a, b, c, d$ thoả mãn $1 \leq a<b<c<d$, ký hiệu: $T(a, b, c, d)=[(x, y, z, t) \subset \mathbb{N}^* \mid 1 \leq x<y<z<t, x \leq a, y \leq b, z \leq c, t \leq d]$.

(a) Tính số phần tử của $T(1,4,6,7)$.

(b) Cho $a=1$ và $b \geq 4$. Gọi $d_1$ là số phần tử của $T(a, b, c, d)$ chứa 1 và không chứa $2 ; d_2$ là số phần tử chứa 1,2 và không chứa $3 ; d_3$ là số phần tử chứa $1,2,3$ và không chứa 4 . Chứng minh rằng $d_1 \geq 2 d_2-d_3$. Đẳng thức xảy ra khi nào ?

Lời giải. (a) Với $T(1,4,6,7)$, ta có $x \leq 1$ nên $x=1$. Khi đó ta có $2 \leq y \leq 4$ hay $y \in{2,3,4}$. Xét các khả năng sau:

  • Nếu $y=2$ thì $3 \leq z \leq 6$. Với mỗi giá trị của $z$, ta có thể thu được $7-z$ giá trị của $t$ nên ta có 10 bộ số.

  • Nếu $y=3$, tương tự ta có 6 bộ số.

  • Nếu $y=4$, tương tự ta có 3 bộ số.

Vậy có tất cả 19 bộ số trong $T(1,4,6,7)$.

(b) Đặt các tập hợp sau:

$\quad\quad\quad\quad\quad\quad\quad \left\{\begin{array}{l}T_1={(1, y, z, t) \mid 3 \leq y \leq b, y<z \leq c, z<t \leq d} \\ T_2={(1,2, z, t) \mid 4 \leq z \leq c, z<t \leq d} \\ T_3={(1,2,3, t) \mid 5 \leq t \leq d}\end{array}\right.$

Ta có $d_3=\left|T_3\right|=d-4$ và

$\quad\quad\quad\quad\quad\quad\quad\quad\quad\quad d_2=\sum_{z=4}^c(d-z)=(c-3) d+\frac{(c+4)(c-3)}{2}$

Tiếp theo ta tính $d_1=\left|T_1\right|$. Vì $b \geq 4$ nên $y \geq 3$. Xét các khả năng sau

  • Nếu $y=3$ thì $T(1,3, z, t)=d_2$.

  • Nếu $y=4$ thì $T(1,4, z, t)=\sum_{z=5}^c(d-z)=(c-4) d-\frac{(c+5)(c-4)}{2}$.

Từ đó $d_1 \geq d_2+(c-4) d-\frac{(c+5)(c-4)}{2}$. Do đó, kết hợp với việc tính được giá trị của $d_2$, khi cộng theo vế thì $d_1+d_3-2 d_2 \geq 0$.

Vậy $d_1 \geq 2 d_2-d_3$. Đẳng thức xảy ra khi và chỉ khi $b=4$.

Nhận xét. Ngoài lời giải khá “đại số” phía trên, có một lời giải khác cho ý sau của bài toán sử dụng song ánh:

  • Điểm mấu chốt là phân rã $T_1, T_2, T_3$ thành các nhóm thích hợp và thiết lập được đơn ánh giữa chúng. Với các tập $T_1, T_2, T_3$ định nghĩa như trên, ta viết $T_1$ thành $A \cup B \cup C$ có giao đôi một khác rỗng, trong đó

$\quad\quad\quad\quad\quad\quad \left\{\begin{array}{l}A={(1,3,4, t) \mid 5 \leq t \leq d} \\ B={(1,3, z, t) \mid 5 \leq z \leq c, z<t \leq d} \\ C={(1, y, z, t) \mid 4 \leq y \leq b, y<z \leq c, z<t \leq d}\end{array}\right.$

  • Dễ kiểm chứng rằng có song ánh từ $A$ vào $T_3$ nên $|A|=\left|T_3\right|=d_3$.

  • Xét $D={(1,4, z, t) \mid 5 \leq z \leq c, z<t \leq d}$. Dễ kiểm chứng rằng $D \subset C$ và có song ánh từ $D$ vào $B$ nên $|D|=|B|$.

  • Ta có $A \cup B={(1,3, z, t) \mid 4 \leq z \leq c, z<t \leq d}$. Dễ kiểm chứng rằng có song ánh từ $A \cup B$ vào $T_2$ nên $|A \cup B|=\left|T_2\right|=d_2$. Chú ý rằng $A \cap B=\varnothing$ nên $|A|+|B|=d_2$ hay $|B|=d_2-d_3$. Từ đó ta có:

$\quad\quad\quad\quad\quad d_1=|A|+|B|+|C| \geq|A|+|B|+|D|=d_3+2|B|$

Vậy $d_1 \geq d_3+2\left(d_2-d_3\right)=2 d_2-d_3$. Đẳng thức xảy ra khi và chỉ khi $b=4$.

Bài 7. Trong một hệ thống máy tính, một máy tính bất kỳ có kết nối trực tiếp với ít nhất $30 \%$ máy tính khác của hệ thống. Hệ thống này có một chương trình cảnh báo và ngăn chặn khá tốt, do đó khi một máy tính bị virus, nó chỉ có đủ thời gian lây cho các máy tính được kết nối trực tiếp với nó. Chứng minh rằng dù vậy, kẻ tấn công vẫn có thể chọn hai máy tính của hệ thống mà nếu thả virus vào hai máy đó, ít nhất $50 \%$ máy tính của hệ thống sẽ bị nhiễm virus.

Lời giải Trước hết ta chứng minh bổ đề sau: Xét một tập con $S$ bất kỳ của tập các máy tính $X$, khi đó tồn tại 1 máy tính của hệ thống kết nối trực tiếp với ít nhất $30 \%$ máy tính của $S$.

Thật vậy, xét các cặp $(s, x)$ với $s \in S, x \in X$ và $(s, x)$ kết nối trực tiếp với nhau. Khi đó, nếu tính theo $s$ thì số cặp như vậy sẽ không ít hơn $\frac{3}{10}|S||X|$. Do đó nếu tính theo $x$ thì sẽ phải tồn tại máy tính $x$ kết nối trực tiếp với ít nhất $\frac{3}{10}|S|$.

Quay trở lại bài toán,

Giả sử hệ thống có $n$ máy tính. Xét máy tính $A$ bất kỳ. Gọi $S$ là tập hợp các máy tính không kết nối trực tiếp với $A$. Nếu $S=\varnothing$ thì kết quả bài toán là hiển nhiên. Nếu $S \neq \varnothing$ thì theo bổ đề, tồn tại máy tính $B$ kết nối trực tiếp với ít nhất $30 \%$ máy tính trong $S$. Ta chứng minh hai máy tính $A$ và $B$ thỏa mãn yêu cầu bài toán.

Thật vậy, giả sử $A$ kết nối trực tiếp với $k$ máy tính khác. Khi đó, theo cách chọn, $A$ và $B$ sẽ kết nối trực tiếp với ít nhất

$\quad\quad\quad\quad\quad k+0,3(n-k)=0,7 k+0,3 n \geq 0,7 \cdot 0,3 n+0,3 n=0,51 n .$

Từ đây ta có được kết luận của bài toán.

Bài 8 . Cho tam giác $A B C$ nhọn. Đường tròn $(I)$ có tâm $I$ thuộc cạnh $B C$ và tiếp xúc với các cạnh $A B, A C$ lần lượt tại $E, F$. Lấy $M, N$ bên trong tứ giác $B C E F$ sao cho $E F N M$ nội tiếp $(I)$ và các đường thẳng $M N, E F, B C$ đồng quy. Gọi $M F$ cắt $N E$ tại $P, A P$ cắt $B C$ tại $D$.

(a) Chứng minh rằng $A, D, E, F$ cùng thuộc một đường tròn.

(b) Lấy trên các đường thẳng $B N, C M$ các điểm $H, K$ sao cho $\angle A C H=$ $\angle A B K=90^{\circ}$. Gọi $T$ là trung điểm $H K$. Chứng minh rằng $T B=T C$.

Lời giải. (a) Ta sẽ chứng minh rằng $A D \perp B C$. Gọi $X$ là điểm đồng quy của $E F, M N, B C$. Do $A E, A F$ tiếp xúc với $(I)$ nên $E F$ là đường đối cực của $A$ đối với (I). Ta có $X \in E F$ nên theo định lý La Hire, điểm $A$ sẽ nằm trên đường đối cực của $X$ đối với đường tròn $(I)$.

Lại có $P$ là giao điểm của $E N, F M$ nên $P$ nằm trên đường đối cực của $X$ đối với $(I)$. Vì thế nên $A P$ là đường đối cực của $X$ đối với $(I)$ hay $A P \perp B C$. Do đó

$\quad\quad\quad\quad\quad\quad\quad\quad\quad\quad \angle A D I=\angle A E I=\angle A F I=90^{\circ} .$

Vậy $A, D, E, F$ cùng thuộc một đường tròn.

(b) Gọi $S$ là giao điểm của $B N, C M$. Xét hai tam giác $P E F, S B C$ có $P E$ cắt $S B$ tại $N, P F$ cắt $S C$ tại $M, E F$ cắt $B C$ tại $X$ và $X, M, N$ thẳng hàng. Theo định lý Desargues thì $P S, E B, F C$ đồng quy. Mặt khác $E B$ cắt $F C$ tại $A$ nên $A, P, S$ thẳng hàng, dẫn đến $S \in A D$.

Tiếp theo ta sẽ chứng minh rằng $\angle B A K=\angle C A H$. Áp dụng định lý Ceva dạng lượng giác cho tam giác $A B C$ với:

  • Các đường thẳng $A D, B H, C K$ đồng quy:

$\quad\quad\quad\quad\quad\quad\quad\quad\quad\quad \frac{\sin \angle D A B}{\sin \angle D A C} \cdot \frac{\sin \angle H B C}{\sin \angle H B A} \cdot \frac{\sin \angle K C A}{\sin \angle K C B}=1$

  • Các đường thẳng $A H, B H, C H$ đồng quy:

$\quad\quad\quad\quad\quad\quad\quad\quad\quad\quad \frac{\sin \angle H A B}{\sin \angle H A C} \cdot \frac{\sin \angle H B C}{\sin \angle H B A} \cdot \frac{\sin \angle H C A}{\sin \angle H C B}=1$

  • Các đường thẳng $A K, B K, C K$ đồng quy:

$\quad\quad\quad\quad\quad\quad\quad\quad\quad\quad \frac{\sin \angle K A B}{\sin \angle K A C} \cdot \frac{\sin \angle K B C}{\sin \angle K B A} \cdot \frac{\sin \angle K C A}{\sin \angle K C B}=1$

Chú ý rằng do các góc vuông và góc bù nhau nên ta có

$\quad\quad\quad\quad\quad\quad\quad\quad\quad\quad \frac{\sin \angle H A C}{\sin \angle H A B}=\frac{\sin \angle K A B}{\sin \angle K A C}$

Từ đó sử dụng công thức cộng cho mẫu thức và biến đổi thì:

$\quad\quad\quad\quad\quad\quad\quad\quad\quad\quad \tan \angle H A C=\tan \angle K A B$

Dẫn đến $\angle H A C=\angle K A B$. Cuối cùng, ta sẽ chứng minh $T B=T C$.

Gọi $U, V$ lần lượt là trung điểm của các đoạn $A K, A H$. Ta có:

$\quad\quad\quad\quad\quad\quad\quad\quad\quad\quad U B=\frac{A K}{2}=V T, U T=\frac{A H}{2}=V C .$

Đồng thời, ta cũng có:

$\quad\quad\quad\quad\quad\quad \angle B U T=\angle B U A-\angle A U T=\angle A V C-\angle A V T=\angle T V C$

Do đó $\Delta B U T=\Delta T V C$ (c.g.c), vậy nên $T B=T C$.

Nhận xét. Để chứng minh $\angle H A C=\angle K A B$, cũng là mấu chốt của lời giải trên, ta có thể dùng bổ đề sau:

Cho tam giác $A B C$ có hai điểm $P, Q$ sao cho $A P, A Q$ đẳng giác trong góc $A$. Gọi $X$ là giao điểm của $B P, C Q$ và $Y$ là giao điểm của $B Q, C P$. Khi đó, ta cũng có $A X, A Y$ đẳng giác trong góc $A$.

 

 

 

 

 

 

 

 

 

 

 

 

 

 

 

 

 

 

 

 

 

 

 

 

 

 

 

 

 

 

 

 

 

 

 

 

 

 

 

 

 

 

 

 

 

 

 

 

 

 

 

 

 

 

 

 

 

 

 

 

 

ĐỀ THI VÀO LỚP 10 CHUYÊN TOÁN TP.HCM NĂM 2017

Bài 1. (a) Cho các số thực $a, b, c$ sao cho $a+b+c=3, a^{2}+b^{2}+c^{2}=29$ và $a b c=11$. Tính $a^{5}+b^{5}+c^{5}$

(b) Cho biểu thức $A=(m+n)^{2}+3 m+n$ với $m, n$ là các số nguyên dương. Chứng minh rằng nếu $A$ là một số chính phương thì $n^{3}+1$ chia hết cho $m$.

Bài 2. (a) Giải hệ phương trình: $2(x+2) \sqrt{3 x-1}=3 x^{2}-7 x-3$

(b) Giải hệ phương trình: $\left\{\begin{array}{l}x+\frac{1}{y}-\frac{10}{x}=-1 \\ 20 y^{2}-x y-y=1\end{array}\right.$

Bài 3. Cho tam giác $A B C$ có $A B<A C<B C$. Trên các cạnh $B C, A C$ lần lượt lấy các điểm $M, N$ sao cho $A N=A B=B M$. Các đường thẳng $A M$ và $B N$ cắt nhau tại $\mathrm{K}$. Gọi $H$ là hình chiếu của $K$ lên $\mathrm{AB}$. Chứng minh rằng:

(a) Tâm đường tròn nội tiếp tam giác $A B C$ nằm trên $K H$.

(b) Các đường tròn nội tiếp các tam giác $A C H$ và $B C H$ tiếp xúc với nhau.

Bài 4. Cho $x, y$ là 2 số thực dương. Tìm giá trị nhỏ nhất của biếu thức:

$P=\frac{16 \sqrt{x y}}{x+y}+\frac{x^{2}+y^{2}}{x y}$

Bài 5. Cho tam giác $A B C$ có góc $B$ tù. Đường tròn $(O)$ nội tiếp tam giác $A B C$ tiếp xúc với các cạnh $A B, C A, B C$ lần lượt tại $L, H, J$.

(a) Các tia $B O, C O$ cắt $L H$ lần lượt tại $M, N$. Chứng minh 4 điểm $B, C, M, N$ cùng thuộc một đường tròn.

(b) Gọi $d$ là đường thẳng qua $O$ và vuông góc với $A J$; $d$ cắt $A J$ và đường trung trực của cạnh $B C$ lần lượt tại $D$ và $F$. Chứng minh 4 điểm $B, D, F, C$ cùng thuộc một đường tròn.

Bài 6. Trên một đường tròn có 9 điểm phân biệt, các điểm này được nối với nhau bởi các đoạn thẳng màu xanh hoặc màu đỏ. Biết rằng mỗi tam giác tạo bởi 3 trong 9 điểm chứa ít nhất một cạnh màu đỏ. Chứng minh rằng tồn tại 4 điểm sao cho 6 đoạn thẳng nối chúng đều có màu đỏ.

LỜI GIẢI

Bài 1.

a) Cho các số thực $a, b, c$ sao cho $a+b+c=3, a^{2}+b^{2}+c^{2}=29$ và $a b c=11$. Tính $a^{5}+b^{5}+c^{5}$

b) Cho biểu thức $A=(m+n)^{2}+3 m+n$ với $m, n$ là các số nguyên dương. Chứng minh rằng nếu $A$ là một số chính phương thì $n^{3}+1$ chia hết cho $m$.

Lời giải.

a) Đặt $S_{2}=a^{2} b^{2}+b^{2} c^{2}+a^{2} c^{2} ; S_{3}=a^{3}+b^{3}+c^{3} ; S_{5}=a^{5}+b^{5}+c^{5}$

  • $(a+b+c)^{2}=a^{2}+b^{2}+c^{2}+2(a b+b c+a c) $

$\quad\quad\Rightarrow 9=29+2(a b+b c+a c) $

$\quad\quad\Rightarrow a b+b c+a c=-10 $

  • $(a+b+c)\left(a^{2}+b^{2}+c^{2}\right)=a^{3}+b^{3}+c^{3}+a^{2} b+a b^{2}+b^{2} c+b c^{2}+a^{2} c+a c^{2} $

$\quad\quad\Rightarrow 3.29=S_{3}+a b(a+b)+b c(b+c)+a c(a+c) $

$\quad\quad\Rightarrow 87=S_{3}+(a+b+c)(a b+b c+a c)-3 a b c $

$\quad\quad \Rightarrow S_{3}=87-3 \cdot(-10)+3.11=150 $

  • $(a b+b c+a c)^{2}=a^{2} b^{2}+b^{2} c^{2}+a^{2} c^{2}+2 a b c(a+b+c) $

$\quad\quad \Rightarrow 100=S_{2}+2.11 .3 \Rightarrow S_{2}=34 $

  • $\left(a^{2}+b^{2}+c^{2}\right)\left(a^{3}+b^{3}+c^{3}\right)=a^{5}+b^{5}+c^{5}+a^{2} b^{3}+a^{3} b^{2}+b^{2}c^{3}+b^{3}c^{2}+a^{3} c^{2}+a^{2} c^{3} $

$\quad\quad\Rightarrow 29.150=S_{5}+a^{2} b^{2}(a+b)+b^{2} c^{2}(b+c)+a^{2} c^{2}(a+c) $

$\quad\quad \Rightarrow 29.150=S_{5}+(a+b+c)\left(a^{2} b^{2}+b^{2} c^{2}+a^{2} c^{2}\right)-a b c(a b+b c+a c) $

$\quad\quad\Rightarrow S_{5}=29.150-3.34+11 .(-10)=4138$

Nhận xét thêm: Trên thực tế, phương trình bậc 3 nhận $a, b, c$ làm nghiệm chỉ có một nghiệm thực, mà đề thi cho 3 số $a, b, c$ thực.

b) Do $m, n$ là số nguyên dương nên ta có: $A=(m+n)^{2}+3 m+n>(m+n)^{2}$ $A=(m+n)^{2}+3 m+n=m^{2}+n^{2}+3 m+n+2 m n<m^{2}+n^{2}+4+4 m+4 n+$ $2 m n=(m+n+2)^{2}$

Mà $\mathrm{A}$ là số chính phương nên $A=(m+n+1)^{2}$

$\Rightarrow(m+n)^{2}+3 m+n=(m+n+1)^{2} $

$\Rightarrow m^{2}+n^{2}+2 m n+3 m+n=m^{2}+n^{2}+1+2 m+2 n+2 m n $

$\Rightarrow m=n+1 .$

Lại có $n^{3}+1=(n+1)\left(n^{2}-n+1\right) \vdots(n+1) \Rightarrow n^{3}+1 \vdots m$

Bài 2.

a) Giải hệ phương trình: $2(x+2) \sqrt{3 x-1}=3 x^{2}-7 x-3$

b) Giải hệ phương trình: $\left\{\begin{array}{l}x+\frac{1}{y}-\frac{10}{x}=-1 \\ 20 y^{2}-x y-y=1\end{array}\right.$

Lời giải.

a) Điều kiện: $x \geq \frac{1}{3}$

$2(x+2) \sqrt{3 x-1}=3 x^{2}-7 x-3 $

$\Leftrightarrow x^{2}+4 x+4+2(x+2) \sqrt{3 x-1}+3 x-1=4 x^{2} $

$\Leftrightarrow(x+2+\sqrt{3 x-1})^{2}=(2 x)^{2} $

$\Leftrightarrow x+2+\sqrt{3 x-1}=2 x \quad\left(\text { vì } x \geq \frac{1}{3}\right) $

$\Leftrightarrow \sqrt{3 x-1}=x-2(x \geq 2) $

$\Leftrightarrow 3 x-1=x^{2}-4 x+4 $

$\Leftrightarrow x^{2}-7 x+5=0\Leftrightarrow\left[\begin{array}{rl}x & =\frac{7+\sqrt{29}}{2}(n) \\ x & =\frac{7-\sqrt{29}}{2}(l)\end{array}\right.$

$\left\{\begin{array}{l}x+\frac{1}{y}-\frac{10}{x}=-1 \\ 20 y^{2}-x y-y=1\end{array}\right.$

Điều kiện: $y \neq 0, x \neq 0$

Chia 2 vế của (2) cho $y$ ta được:

$20 y-x-1=\frac{1}{y}$

$\Rightarrow 20 y-x=\frac{1}{y}+1$

Mà $\frac{1}{y}+1=\frac{10}{x}-x$ nên

$20 y-x=\frac{10}{x}-x$

$\Rightarrow x y=\frac{1}{2}$

Thay vào (2) ta được:

$20 y^{2}-\frac{1}{2}-y=1 \Rightarrow 40 y^{2}-2 y-3=0 \Rightarrow\left[\begin{array}{l}y=\frac{3}{10} \Rightarrow x=\frac{5}{3} \\ y=-\frac{1}{4} \Rightarrow x=-2\end{array}\right.$

Vậy nghiệm của hệ phương trình là $(x ; y) \in[(-2 ;-\frac{1}{4}),(\frac{5}{3} ; \frac{3}{10})]$

Bài 3. Cho tam giác $A B C$ có $A B<A C<B C$. Trên các cạnh $B C, A C$ lần lượt lấy các điểm $M, N$ sao cho $A N=A B=B M$. Các đường thẳng $A M$ và $B N$ cắt nhau tại $\mathrm{K}$. Gọi $H$ là hình chiếu của $K$ lên $\mathrm{AB}$. Chứng minh rằng:

a) Tâm đường tròn nội tiếp tam giác $A B C$ nằm trên $K H$.

b) Các đường tròn nội tiếp các tam giác $A C H$ và $B C H$ tiếp xúc với nhau.

Lời giải.

a) Gọi $I$ là tâm đường tròn nội tiếp tam giác $A B C$.

Tam giác $A B N$ cân tại $A$ nên phân giác góc $B A C$ cũng là đường cao, suy ra $A I \perp B N$.

Tam giác $A B M$ cân tại $B$ nên phân giác góc $A B C$ cũng là đường cao, suy ra $B I \perp A M$.

Suy ra $I$ là trực tâm tam giác $A B K$, mà $K H \perp A B$ nên $K, I, H$ thẳng hàng.

Vậy tâm đường tròn nội tiếp tam giác $A B C$ nằm trên $K H$.

b) Gọi $D$ là tiếp điểm của $(J)$ với $C H$. TA có $D H=\frac{H A+H C-A C}{2}$.

Gọi $E$ là tiếp điểm của $(L)$ với $C H$. Ta có $H E=\frac{H C+H B-B C}{2}$

Gọi $P$ và $Q$ lần lượt là tiếp điểm của $(I)$ với $A C$ và $B C$ $H D-H E=\frac{H A-A C-H B+B C}{2}=\frac{B C-A C+H A-H B}{2}$ $=\frac{B Q+C Q-A P-C P+H A-H B}{2}=0$ (vì $H$ là tiếp điểm của $(I)$ với $A B$ nên $A H=A P, B H=B Q$

Do đó $D$ trùng $E$. nên hai đường tròn $(J)$ và $(L)$ tiếp xúc nhau

Bài 4. Cho $x, y$ là 2 số thực dương. Tìm giá trị nhỏ nhất của biếu thức:

$P=\frac{16 \sqrt{x y}}{x+y}+\frac{x^{2}+y^{2}}{x y}$

Lời giải.  $P=\frac{16 \sqrt{x y}}{x+y}+\frac{x^{2}+y^{2}}{x y} $

$=\frac{16 \sqrt{x y}}{x+y}+\frac{(x+y)^{2}}{x y}-2 $

$=\frac{8 \sqrt{x y}}{x+y}+\frac{8 \sqrt{x y}}{x+y}+\frac{(x+y)^{2}}{x y}-2 $

$\geq 3 \sqrt[3]{64}-2=10$

Dấu ” $=$ “xảy ra khi và chỉ khi $x=y$ Vậy $P_{\min }=10 \Leftrightarrow x=y$

Bài 5. Cho tam giác $A B C$ có góc $B$ tù. Đường tròn $(O)$ nội tiếp tam giác $A B C$ tiếp xúc với các cạnh $A B, C A, B C$ lần lượt tại $L, H, J$.

a) Các tia $B O, C O$ cắt $L H$ lần lượt tại $M, N$. Chứng minh 4 điểm $B, C, M, N$ cùng thuộc một đường tròn.

b) Gọi $d$ là đường thẳng qua $O$ và vuông góc với $A J ; d$ cắt $A J$ và đường trung trực của cạnh $B C$ lần lượt tại $D$ và $F$. Chứng minh 4 điểm $B, D, F, C$ cùng thuộc một đường tròn.

Lời giải.

a) $N, O$ nằm trên đường trung trực của đoạn $H J$ nên $N H=N J, O H=O J$ $\Rightarrow \triangle N H O=\triangle N J O \Rightarrow \angle N H O=\angle N J O$.

Mà $\angle N H O=\angle N L O$ nên $\angle N J O=\angle N L O \Rightarrow L N O J$ nội tiếp.

Lại có $B L O J$ nội tiếp nên 5 điểm $B, J, O, N, L$ cùng nằm trên một đường tròn.

Suy ra $B N O J$ là tứ giác nội tiếp, suy ra $\angle B N O=90^{\circ}$.

$M, O$ nằm trên đường trung trực của đoạn $L J$ nên $M L=M J, O L=O J \Rightarrow$ $\triangle M O L=\triangle M O J \Rightarrow \angle O L M=\angle O J M$

Mà $\angle O L H=\angle O H L$ nên $\angle O H L=\angle O J M \Rightarrow O H M J$ nội tiếp.

Lại có $O H C J$ nột tiếp nên $O, H, M, C$, $J$ cùng thuộc một đường tròn nên $O M H C$ nội tiếp $\Rightarrow \angle O M C=\angle O H C=90^{\circ}$

$\angle B N C=\angle B M C=90^{\circ} \Rightarrow B M N C$ nội tiếp.

b) Gọi $E$ là giao điểm của $\mathrm{MN}$ và $B C$. Ta chứng minh $O E \perp A J$.

Ta có $O K . O A=O H^{2}=O J^{2}$, suy ra tam giác $O K J$ và $O J A$ đồng dạng, suy ra $\angle O K J=\angle O J A$.

Mặt khác tứ giác $O K E J$ nội tiếp nên $\angle O K J=\angle O E J$.

Do đó $\angle O J A=\angle O E J$, suy ra $O E \perp A J$. Khi đó $O E$ cắt $A J$ tại $D$ và cắt trung trực $B C$ tại $F$.

Xét tam giác $T B C$ chứng minh được $M, N, Q, J$ cùng thuộc đường tròn.

Ta có $E D . E F=E J . E Q=E M . E N=E B . E C$

Suy ra $B D F C$ nội tiếp.

Bài 6. Trên một đường tròn có 9 điểm phân biệt, các điểm này được nối với nhau bởi các đoạn thẳng màu xanh hoặc màu đỏ. Biết rằng mỗi tam giác tạo bởi 3 trong 9 điểm chứa ít nhất một cạnh màu đỏ. Chứng minh rằng tồn tại 4 điểm sao cho 6 đoạn thẳng nối chúng đều có màu đỏ.

Lời giải. Giả sử không tồn tại 4 điểm nào sao cho 6 đoạn thẳng nối chúng đều có màu đỏ.

$-$ Nếu tồn tại một điểm nối ít nhất 4 điểm khác để tạo thành đoạn thẳng màu xanh, giả sử $A$ nối với $B, C, D, E$ tạo thành đoạn màu xanh, khi đó:

$-$ Nếu có hai trong 4 điểm $B, C, D, E$ nối với nhau tạo thành đoạn màu xanh thì mâu thuẫn

$-$ Nếu 4 điểm này đôi một không nối với nhau tạo thành đoạn màu xanh thì cũng mâu thuẫn.

$-$ Nếu mỗi điểm chỉ nối tối đa với 3 điểm khác để tạo thành đoạn màu xanh. Giả sử $A$ nối với $B, C, D$ tạo thành đoạn màu xanh thì $B C, C D, B D$ màu đỏ và còn lại 5 điểm $M, N, P, Q, R$ nối với $A$ tạo thành đoạn màu đỏ.

$-$ Nếu trong 5 điểm đó, điểm nào cũng nối với 2 điểm trong đó tạo thành đoạn màu xanh, khi đó, mỗi điểm chỉ tạo thêm được đoạn màu xanh với 1 điểm nữa khác 5 điểm $M, N, P, Q, R$. Mà 5 điểm này đều phải tạo với một trong 3 điểm $B, C, D$ tạo thành đoạn màu xanh nên có ít nhất hai điểm cùng tạo với 1 điểm trong $B, C, D$ tạo thành đoạn màu xanh, giả sử đoạn $D M$ và $D N$ màu xanh. Khi đó đoạn $M N$ màu đỏ và $M, N$ nối với $B, C$ tạo thành đoạn màu đỏ. Do đó $B, C, M, N$ là 4 điểm khi nối nhau tạo thành 6 đoạn màu đỏ nên mâu thuẫn.

$-$ Nếu trong 5 điểm $M, N, P, Q$, $R$ có 1 điểm nối với 1 trong 4 điểm còn lại tạo thành đoạn màu xanh, giả sử $M N$ màu xanh. Khi đó $P M, Q M, R M$ màu đỏ. Nếu $P Q, Q R, P R$ đều màu xanh thì mâu thuẫn, nếu 1 trong 3 đoạn màu đỏ, giả sử $P Q$ màu đỏ. Khi đó $A, M, P, Q$ là 4 điểm khi nối nhau tạo thành 4 đoạn màu đỏ. (mâu thuẫn).

$-$ Nếu mỗi điểm chỉ nối tối đa với 2 điểm khác tạo thành đoạn màu xanh, giả sử $A B$, $A C$ màu xanh. Khi đó 6 điểm còn lại $M, N, P, Q, R, T$ khi nối $A$ tạo thành màu đỏ. Giả sử $M B$ màu xanh thì $M$ nối được với 1 điểm nữa trong 5 điểm còn lại tạo thành đoạn màu xanh, giả sử $M N$ màu xanh. Khi đó $M P, M Q, M R, M T$ màu đỏ. Trong 4 điểm $P, Q, R, T$ tồn tại 2 điểm nối nhau tạo thành màu đỏ, giả sử $P Q$ màu đỏ. Khi đó $A, M, P, Q$ là 4 điểm nối nhau tạo thành 6 đoạn màu đỏ. Giả sử $M B, M C$ đều màu đỏ thì $M$ tạo được với tối đa 2 điểm nữa trong 5 điểm còn lại thành đoạn màu xanh. Giả sử $M N, M P$ màu xanh thì $M Q, M R, M T$ màu đỏ, trong 3 điểm $Q, R, T$ tồn tại 2 điểm nối nhau tạo thành đoạn màu đỏ, giả sử $Q R$ màu đỏ thì $A, M, Q, R$ là 4 điểm nối nhau tạo thành 6 đoạn màu đỏ (mâu thuẫn).

Vậy tồn tại 4 điểm sao cho 6 đoạn thẳng nối chúng đều có màu đỏ.

 

 

 

 

 

 

 

 

 

 

 

 

 

 

 

 

 

 

 

 

 

 

 

 

 

 

 

 

 

 

 

 

 

 

 

Đề thi và đáp án tuyển sinh vào lớp 10 PTNK không chuyên 2017

Đề thi vào lớp 10 trường Phổ thông Năng khiếu năm 2017

Bài 1. Biết $a$, $b$ là các số dương, $a \ne b$ và

$$\left( \dfrac{\left( a+ 2b \right) ^2 – \left( b+ 2a \right) ^2 }{a+ b} \right) : \left( \dfrac{\left( a\sqrt{a} + b\sqrt{b} \right) \left( a\sqrt{a} – b\sqrt{b } \right) }{a-b} – 3ab \right) = 3$$

Tính $S= \dfrac{1+2ab}{a^2 + b^2}$

Giải

$\left( \dfrac{\left( a+ 2b \right) ^2 – \left( b+ 2a \right) ^2 }{a+ b} \right) : \left( \dfrac{\left( a\sqrt{a} + b\sqrt{b} \right) \left( a\sqrt{a} – b\sqrt{b } \right) }{a-b} – 3ab \right) = 3$

$\Leftrightarrow \dfrac{a^2 + 4ab + 4b^2 – b^2 – 4ab – 4a^2}{a+b} : \left( \dfrac{a^3 – b^3}{a-b} – 3ab \right) =3$

$\Leftrightarrow \dfrac{3(b-a)(a+b)}{a+b}:\left( a^2 -2ab + b^2 \right) =3 $

$\Leftrightarrow \dfrac{3(b-a)}{(a-b)^2}=3 \Leftrightarrow a-b=-1 \Rightarrow a= b-1 $

Thay $a=b-1$ vào $S$, ta được:

$S= \dfrac{1+ 2ab}{a^2 + b^2} = \dfrac{1+ 2(b-1)b}{(b-1)^2 + b^2} = \dfrac{1+ 2b^2 – 2b}{2b^2 -2b +1}=1$

Bài 2.

a) Giải phương trình $\left( x^2 – 6x + 5 \right) \left( \sqrt{x-2} – x + 4 \right) =0$.

b) Giải hệ phương trình $\left\{ \begin{array}{l} \sqrt{x}\left( \sqrt{x+ 2y} -3 \right) =0 \\ x^2 – 6xy – y^2 = 6  \end{array} \right. $

Giải

a) Điều kiện: $x \ge 2$

$\left( x^2 – 6x + 5 \right) \left( \sqrt{x-2} – x + 4 \right) =0 $

$\Leftrightarrow \left[ \begin{array}{l} x^2 – 6x + 5 = 0 \;\; (1)\\ \sqrt{x-2} – x+ 4=0 \;\; (2) \end{array} \right.$

$(1) \Leftrightarrow \left[ \begin{array}{l} x=1 \;\; (l) \\ x= 5 \;\; (n) \end{array}\right. $

$(2) \Leftrightarrow \sqrt{x-2} = x-4 \;\; (x \ge 4)$

$\Leftrightarrow x-2 = x^2 – 8x + 16 $

$\Leftrightarrow x^2 – 9x + 18 = 0 \Leftrightarrow \left[ \begin{array}{l} x=3 \;\; (l)\\ x=6 \;\; (n) \end{array} \right. $

Vậy $S= \left\{5;6 \right\}$

b) Điều kiện $x \ge 0$, $x+ 2y \ge 0$

$\left\{ \begin{array}{l} \sqrt{x}\left( \sqrt{x+ 2y} -3 \right) =0 \;\; (1) \\ x^2 – 6xy – y^2 = 6 \;\; (2) \end{array} \right. $

$(1) \Leftrightarrow \left[ \begin{array}{l} x=0 \\ \sqrt{x+2y} -3=0 \end{array} \right. $

  •  Nếu $x=0$, thay vào (2) ta được: $-y^2 = 6$ (Vô nghiệm)
  •  Nếu $\sqrt{x+2y} -3 =0 $

$\Leftrightarrow x+2y = 9 \Leftrightarrow x= 9-2y$

Thay $ x= 9-2y$ vào (2), ta được:

$(9-2y)^2 – 6(9-2y)y – y^2 = 6 $

$\Leftrightarrow 4y^2 – 26y +81 – 54y + 12y^2 -y^2 = 6$

$\Leftrightarrow 15y^2 – 90 y + 75 =0 $

$\Leftrightarrow \left[ \begin{array}{l} y=1 \Rightarrow x=7 \;\; (n)\\ y=5 \Rightarrow x= -1 \;\; (l) \end{array} \right. $

Vậy cặp nghiệm của hệ phương trình $(x;y)$ là $(7;1)$

Bài 3. Cho phương trình $(x+m)^2 – 5(x+m) + 6=0$ $(1)$.

a) Chứng minh phương trình $(1)$ luôn có hai nghiệm phân biệt $x_1$, $x_2$ với mọi số thực $m$. Tính $S= \left( x_1 + m \right) ^2 + \left( x_2 + m \right) ^2 + 5 \left( x_1 + x_2 + 2m \right) $.

b) Biết $x_1 < x_2$, tìm $m$ sao cho $x_2 < 1$ và $x_1^2 + 2x_2 = 2(m-1)$.

Giải

a) $(x+m)^2 – 5(x+m) + 6=0$

$\Leftrightarrow (x+m)^2 – 2(x+m) – 3(x+m) +6 = 0 $

$\Leftrightarrow (x+m)(x+m-2) – 3(x+m-2)=0$

$\Leftrightarrow (x+m-2)(x+m-3)=0$

$\Leftrightarrow \left[ \begin{array}{l} x= 2-m \\ x= 3-m \end{array} \right. $

Vì $2-m \ne 3-m$ nên $x_1 \ne x_2$

Vậy phương trình $(1)$ luôn có hai nghiệm phân biệt $x_1$, $x_2$ với mọi số thực $m$.

$S= \left( x_1 + m \right) ^2 + \left( x_2 + m \right) ^2 + 5 \left( x_1 + x_2 + 2m \right) $

Vì $x_1$, $x_2$ có vai trò tương đương trong biểu thức $S$ nên giả sử $x_1 = 2-m$, $x_2 = 3-m$, ta có:

$S= 2^2 + 3^2 + 5(2+3) = 38$

b) $x_1 < x_2$ nên $x_1 = 2-m$, $x_2 = 3-m$.

$x_2<1 \Rightarrow 3-m <1 \Rightarrow m > 2$

$x_1^2 + 2x_2 = 2(m-1) $

$\Rightarrow (2-m)^2 + 2(3-m) = 2(m-1) $

$\Rightarrow m^2 – 4m + 4 + 6 -2m = 2m -2 $

$\Rightarrow m^2 -8m + 12 = 0 $

$\Rightarrow \left[ \begin{array}{l} m= 6 \;\; (n)\\ m=2 \;\; (l) \end{array} \right. $

Vậy $m=6$

Bài 4.

a) Nam kể với Bình rằng ông của Nam có một mảnh đất hình vuông $ABCD$ được chia thành bốn phần; hai phần (gồm các hình vuông $AMIQ$ và $INCP$ với $M$, $N$, $P$, $Q$ lần lượt thuộc $AB$, $BC$, $CD$, $DA$) để trồng các loại ra sạch, các phần còn lại trồng hoa. Diện tích phần trồng ra sạch là $1200 \; m^2$ và phần để trồng hoa là $1300 \; m^2$. Bình nói: “Chắc chắn bạn bị nhầm rồi!”. Nam: “Bạn nhanh thật! Mình đã nói nhầm phần diện tích. Chính xác là phần trồng rau sạch có diện tích $1300 \; m^2$, còn lại $1200 \; m^2 $ trồng hoa”. Hãy tính cạnh hình vuông $AMIQ$ (biết $AM < MB$) và giải thích vì sao Bình lại biết Nam bị nhầm ?

b) Lớp $9T$ có $30$ bạn, mỗi bạn dự định đóng góp mỗi tháng $70000$ đồng và sau $3$ tháng sẽ đủ tiền mua tặng cho mỗi em ở “Mái ấm tình thương $X$” ba gói quà (giá tiền các món quà đều như nhau). Khi các bạn đóng đủ số tiền như dự trù thì “Mái ấm tình thương $X$” đã nhận chăm sóc thêm $9$ em và có giá tiền của mỗi món thêm $5\%$ nên chỉ tặng mỗi em hai gói quà. Hỏi có bao nhiêu em của “Mái ấm tình thương $X$” được nhận quà ?

Giải

a) Gọi cạnh của hình vuông $AMIQ$ và $INCP$ lần lượt là $a$ và $b$. ($a<b$ vì $AM < MB$)

Diện tích đất trồng rau là: $a^2+ b^2$

Diện tích đất trồng hoa là $2ab$

Ta có: $\left\{ \begin{array}{l} a^2 + b^2 = 1300 \\ 2ab = 1200 \end{array} \right. $

$\Rightarrow \left\{ \begin{array}{l} (a-b)^2 = 100 \\ ab= 1200 \end{array} \right.$

$\Rightarrow \left\{ \begin{array}{l} a-b= -10 \\ ab= 1200 \end{array}\right. $

$\Rightarrow \left\{ \begin{array}{l} a= 20 \\ b=30 \end{array} \right. $

Vậy cạnh hình vuông $AMIQ$ là $20m$.

Bình biết Nam bị nhầm vì theo Nam nói thì diện tích phần trồng rau là $1200 \; m^2$ nhỏ hơn diện tích phần trồng hoa $1300 \; m^2$. Mà diện tích phần trồng rau là $a^2+b^2$, diện tích phần trồng hoa là $2ab$.

Áp dụng bất đẳng thức Cauchy, ta có $a^2 + b^2 \ge 2ab$ nên diện tích trồng hoa không thể lớn hơn diện tích trồng rau được.

b) Giả sử lúc đầu “Mái ấm tình thương $X$” có $x$ em.

Tổng số tiền các bạn đóng góp được sau $3$ tháng là $3.70000.30 = 6300000$ (đồng)

Giá tiền $1$ món quà dự đinh là $\dfrac{6300000}{3x}= \dfrac{2100000}{x}$

Giá tiền $1$ món quà thực tế là $\dfrac{6300000}{2(x+9)}$

Ta có: $\dfrac{2100000}{x}.1,05= \dfrac{6300000}{2(x+9)} $

$\Leftrightarrow \dfrac{2205}{x} = \dfrac{6300}{2(x+9)}$

$\Leftrightarrow 4410(x+9) = 6300x $

$\Leftrightarrow x= 21$

Vậy lúc đầu “Mái ấm tình thương $X$” có $21$ em. Số em được nhận quà là $30$ em.

Bài 5. Tam giác $ABC$ nội tiếp đường tròn $(T)$ tâm $O$, bán kính $R$; $\angle BAC = 120^\circ $, $\angle ABC = 45^\circ $, $H$ là trực tâm. $AH$, $BH$, $CH$ lần lượt cắt $BC$, $CA$, $AB$ tại $M$, $N$, $P$.

a) Tính $AC$ theo $R$. Tính số đo góc $\angle HPN $ và $\dfrac{MP}{MN}$

b) Dựng đường kính $AD$, $HD$ cắt $(T)$ tại $E$ ($E \ne D$) và cắt $BC$ tại $F$. Chứng minh các điểm $A$, $N$, $H$, $P$, $E$ cùng thuộc một đường tròn và $F$ là trung điểm của $HD$.

c) Chứng minh $AD \bot NP$. Tia $OF$ cắt $(T)$ tại $I$, chứng minh $I$ là tâm đường tròn ngoại tiếp tam giác $HBC$ và $AI$ đi qua trung điểm của $MP$

Giải

a) Ta có $\angle AOC = 2 \angle ABC = 90^\circ$ (góc ở tâm bằng $2$ lần góc nội tiếp cùng chắn $1$ cung).

Suy ra tam giác $OAC$ vuông tại $O$, suy ra $AC^2 = OA^2 + OC^2 = 2R^2 \Rightarrow AC = R\sqrt{2}$.

Tứ giác $BNPC$ có $\angle BNC = \angle BPC =90^\circ$ nên là tứ giác nội tiếp.

Suy ra $\angle HPN = \angle HBC = 90^\circ – \angle ACB = 75^\circ$.

Các tứ giác $ANBM$ và $BNPC$ nội tiếp nên $\angle ANM = \angle ABC = 45^\circ, \angle CNP = \angle PBC = 45^\circ$.

Suy ra $\angle MNP = \angle CNP + \angle CPN = 90^\circ$.

Và $\angle NPB = \angle ACB = \angle APM = 15^\circ$, suy ra $\angle NPM = \angle NPB + \angle APM = 30^\circ$.

Khi đó $\dfrac{MN}{MP} = \sin \angle NPM = \sin 30^\circ = \dfrac{1}{2}$. Suy ra $\dfrac{MP}{MN} = 2$.

b) Ta có $\angle AEF = 90^\circ$ (góc nội tiếp chắn nửa đường tròn).

Ta có $\angle ANH = \angle AEH = \angle APH = 90^\circ$ nên 5 điểm $A, N, H, P, E$ cùng thuộc đường tròn đường kính $AH$.

Ta có $\angle ABD = \angle ACD = 90^\circ$ (góc nội tiếp chắn nửa đường tròn),

suy ra $AB \bot BD$, suy ra $HC || BD$.

Tương tự ta có $HB \bot CN, CD \bot CN$, suy ra $HB || CD$.

Tứ giác $HBDC$ có các cặp cạnh đối song song nên là hình bình hành, suy ra $F$ là trung điểm của $BC$ và $HD$.

b) Ta có $\angle CAD = 45^\circ = \angle CNM$, suy ra $AD || MN$. Mà $MN \bot NP$, suy ra $AD \bot NP$.

Ta có $OF$ là trung trực của $BC$, suy ra $IB = IC$. $\angle BDC = 180^\circ – \angle BAC = 60^\circ$.

Xét tam giác $IOC$ có $\angle IOC = \dfrac{1}{2}\angle BOC = \angle 60^\circ$. Suy ra tam giác $IOC$ đều.

Do đó $IB =IC = IO$. $(1)$

Mặt khác tứ giác $HBOC$ có $\angle BHC + \angle BOC = 60^\circ + 120^\circ = 180^\circ$, suy ra $HBOC$ nội tiếp. $(2)$

Từ $(1)$ và $(2)$ suy ra $I$ là tâm đường tròn ngoại tiếp tam giác $HBC$.

Tam giác $PBC$ có $\angle BPC = 90^\circ, \angle PBC = 45^\circ$ nên là tam giác vuông cân,

suy ra $PB = PC$, suy ra $P$ thuộc trung trực của BC. Do đó $P, O, I$ thẳng hàng và $PI \bot BC$, suy ra $PI||AM$.

Mặt khác ta có $\angle BIH = 2\angle HCB = 90^\circ$, suy ra $HBMI$ nội tiếp, suy ra $\angle IMC = \angle BHI = 45^\circ$.

Suy ra $\angle IMC = \angle PBC = 45^\circ$, suy ra $IM || PA$.

Tứ giác $APIM$ có 2 cặp cạnh đối song song nên là hình bình hành, suy ra $AI$ qua trung điểm của $MP$.

Đề và đáp thi chọn đội dự tuyển PTNK năm học 2017 – 2018

Bài 1. Tìm tất cả các hàm số $f:\mathbb R \rightarrow \mathbb R $ thỏa mãn:
$$f(3f(x)+2y)=10x+f(f(y)+x),\ \forall x,y \in \mathbb R.$$

Bài 2.  Cho tam giác $ABC$ nhọn. Các điểm $D,E$ thay đổi trên cạnh $BC$ sao cho $\angle BAD = \angle CAE$ và $D$ nằm giữa $B,E$. Đường tròn ngoại tiếp các tam giác $ABD,ACE$ cắt nhau tại điểm $M$ khác $A$.
a)  Chứng minh rằng phân giác góc $\angle DME$ luôn đi qua một điểm cố định.
b) Gọi $I$ và $K$ lần lượt là tâm đường tròn nội tiếp của các tam giác $ABM,ACM$. Chứng minh rằng đường thẳng $IK$ luôn đi qua một điểm cố định.

Bài 3.  Cho $n\ge 3$ là số nguyên dương và $2n$ số thực dương $x_1,x_2,\ldots,x_n,y_1,y_2,\ldots,y_n$ thỏa mãn đồng thời các điều kiện sau:
i) $0< x_1y_1<x_2y_2<\ldots< x_ny_n$.
ii) $x_1+x_2+\cdots+x_k \ge y_1+y_2+\cdots+y_k\ \forall k \in {1,2,\ldots,n }$.

Chứng minh rằng $\dfrac{1}{x_1}+\dfrac{1}{x_2}+\ldots+\dfrac{1}{x_n} \le \dfrac{1}{y_1}+\dfrac{1}{y_2}+\ldots+\dfrac{1}{y_n}$.

Bài 4. Cho $S$ là tập hợp khác rỗng có hữu hạn phần tử. Kí hiệu $P(S)$ là tập hợp tất cả các tập con của $S$. Giả sử $f: P(S) \rightarrow P(S)$ là ánh xạ có tính chất sau: với mọi $X,Y \in P(S)$, nếu $X \subset Y$ thì $f(X) \subset f(Y)$.

Chứng minh rằng có tập hợp $T \in P(S)$ để $f(T) = T$.

Giải

Bài 1. 

Thay $y=-\frac{2f(x)}{3}$, ta có
$$f(0)=10x+f\left( f\left( -\frac{2f(x)}{3} \right)+x \right)$$
nên dễ thấy rằng $f$ toàn ánh vì $f(0)-10x$ nhận giá trị trên $\mathbb{R}.$
Giả sử tồn tại $a,b\in \mathbb{R}$ sao cho $f(a)=f(b).$ Thay $y$ lần lượt bởi $a,b,$ ta có
$$f(3f(x)+2a)=f(3f(y)+2b).$$
Vì tính toàn ánh nên có thể thay $3f(x)\to x$, tức là $f(x+2a)=f(x+2b)$ nên $f$ tuần hoàn chu kỳ $T=2(a-b).$ Khi đó, ta có $f(x)=f(x+T),\forall x\in \mathbb{R}.$

Trong đề bài, thay $x\to x+T$ thì
$f(3f(x)+2y)=10x+10T+f(2f(y)-x)$ nên $T=0.$ Suy ra $f$ đơn ánh. Cuối cùng, cho $x=0$ thì
$f(3f(0)+2y)=f(f(y))$ nên
$$3f(0)+2y=f(y)\Leftrightarrow f(y)=2y+\frac{3}{2}f(0),\forall y.$$
Thay $y=0,$ ta có ngay $f(0)=0$ nên $f(y)=2y.$ Thử lại ta thấy thỏa.

Vậy hàm số $f(x)$ cần tìm là $f(x)=2x,\forall x.$

Bài 2.

(a) Do tứ giác $ABDM,ACEM$ nội tiếp nên $\angle DAB=\angle DMB,\angle EAC=\angle EMC$, mà $\angle DAB=\angle EAC$ nên ta có $\angle DMB=\angle EMC.$ Ta sẽ chứng minh bổ đề sau

Bổ đề (hệ thức Steiner) $\frac{DB}{DC}\cdot \frac{EB}{EC}=\frac{A{{B}^{2}}}{A{{C}^{2}}}$.

Thật vậy, kẻ đường tròn$(ADE)$ cắt $AB,AC$ tại $X,Y.$ Khi đó, ta có $DX=EY$ (vì cùng chắn các cung bằng nhau), suy ra $XY\parallel DE$.
Áp dụng phương tích từ các điểm $B,C$ đến đường tròn $(ADE)$ thì
$$BD\cdot BE=BX\cdot BA \text{ và } CE\cdot CD=CY\cdot CA$$
nên suy ra $$\frac{BD\cdot BE}{CE\cdot CD}=\frac{AB}{AC}\cdot \frac{BX}{CY}=\frac{A{{B}^{2}}}{A{{C}^{2}}}.$$
Áp dụng bổ đề này vào tam giác $BMC$ với hai điểm $D,E.$ Ta cũng có $$\frac{DB}{DC}\cdot \frac{EB}{EC}=\frac{M{{B}^{2}}}{M{{C}^{2}}}.$$ Từ đó suy ra $\frac{MB}{MC}=\frac{AB}{AC}$. Gọi $MS$ là phân giác của $\angle DME$ với $S\in BC.$ Suy ra $MS$ cũng là phân giác của góc $\angle BMC.$ Do đó $$\frac{SB}{SC}=\frac{MB}{MC}=\frac{AB}{AC}$$ nên $S$ chính là chân đường phân giác góc $A$ của tam giác $ABC,$ là điểm cố định.

(b) Gọi $J$ là tâm nội tiếp tam giác $ABC$ thì rõ ràng $I\in BJ,K\in CJ.$
Đặt $\angle DAB=\angle EAC=2\alpha ,\angle DAE=2\beta $ thì
$$\frac{IB}{IJ}=\frac{{{S}_{IAB}}}{{{S}_{IAJ}}}=\frac{AI\cdot AB\cdot \sin \alpha }{AI\cdot AJ\cdot \sin \beta }=\frac{AB}{AJ}\cdot \frac{\sin \alpha }{\sin \beta }.$$
Tương tự thì $$\frac{KC}{JC}=\frac{AC}{AJ}\cdot \frac{\sin \alpha }{\sin \beta }$$ nên $\frac{IB}{IJ}:\frac{KC}{KJ}=\frac{AB}{AC}$. Từ đây gọi $T$ là giao điểm của $IK,BC$ thì theo định lý Menelaus cho tam giác $JBC,$ ta có $\frac{TB}{TC}=\frac{AB}{AC}$ nên $T$ là chân phân giác ngoài góc $A$ của tam giác $ABC,$ là điểm cố định.

 

Bài 3. 

Nhắc lại về khai triển Abel, xem như bổ đề:

Bổ đề. Xét 2 dãy số thực ${{a}_{1}},{{a}_{2}},\ldots ,{{a}_{n}}$ và ${{b}_{1}},{{b}_{2}},\ldots ,{{b}_{n}}$. Đặt ${{S}_{k}}={{b}_{1}}+{{b}_{2}}+\cdots +{{b}_{k}}$. Khi đó
$${{a}_{1}}{{b}_{1}}+{{a}_{2}}{{b}_{2}}+\cdots +{{a}_{n}}{{b}_{n}}=({{a}_{1}}-{{a}_{2}}){{S}_{1}}+({{a}_{2}}-{{a}_{3}}){{S}_{2}}+\cdots +({{a}_{n-1}}-{{a}_{n}}){{S}_{n}}+{{a}_{n}}{{S}_{n}}.$$
Trở lại bài toán đã cho, chuyển vế và quy đồng, ta cần có
$$\frac{{{x}_{1}}-{{y}_{1}}}{{{x}_{1}}{{y}_{1}}}+\frac{{{x}_{2}}-{{y}_{2}}}{{{x}_{2}}{{y}_{2}}}+\cdots +\frac{{{x}_{n}}-{{y}_{n}}}{{{x}_{n}}{{y}_{n}}}>0.$$
Đặt ${{b}_{k}}={{x}_{k}}-{{y}_{k}}$ và ${{a}_{k}}=\frac{1}{{{x}_{k}}{{y}_{k}}}$ với $1\le k\le n$, ta cần chứng minh
$${{a}_{1}}{{b}_{1}}+{{a}_{2}}{{b}_{2}}+\cdots +{{a}_{n}}{{b}_{n}}>0.$$
Chú ý rằng $${{S}_{k}}={{b}_{1}}+{{b}_{2}}+\cdots +{{b}_{k}}=({{x}_{1}}+{{x}_{2}}+\cdots +{{x}_{k}})-({{y}_{1}}+{{y}_{2}}+\cdots +{{y}_{k}})>0$$ đúng theo ii).
Ngoài ra, dãy ${{a}_{k}}$ là dãy giảm nên ${{a}_{1}}-{{a}_{2}},{{a}_{2}}-{{a}_{3}},\ldots ,{{a}_{n-1}}-{{a}_{n}}>0$. Từ đó, áp dụng khai triển Abel ở trên, ta có ngay đpcm.

 

Bài 4.

Nếu như $f(S)=S$ thì ta có đpcm.

Giả sử rằng $f(S)\ne S$. Ta đặt $f(S)={{S}_{1}}$ là một tập con thực sự của $S.$ Khi đó vì ${{S}_{1}}\subset S$ nên ta phải có $f({{S}_{1}})\subset f(S)\Rightarrow f({{S}_{1}})\subset {{S}_{1}}$.

Nếu $f({{S}_{1}})={{S}_{1}}$ thì ta cũng có đpcm nên giả sử $f({{S}_{1}})={{S}_{2}}\ne {{S}_{1}}$ và ${{S}_{2}}\subset {{S}_{1}}.$

Tiếp tục như thế, ta thấy rằng với mỗi số nguyên dương $k$ thì hoặc là $f({{S}_{k}})={{S}_{k}}$ hoặc $f({{S}_{k}})={{S}_{k+1}}$ là tập con thực sự của ${{S}_{k}}.$ Và nếu như không có trường hợp thứ nhất xảy ra thì quá trình này lặp lại vô hạn lần, và sinh ra vô hạn tập con thực sự của tập hữu hạn $S$ ban đầu. Đây là điều vô lý.

Vậy nên luôn tồn tại $T \in P(S)$ để cho $f(T)=T.$

Đáp án đề thi HK1 lớp 11 trường PTNK năm học 2017 – 2018

Bài 1. Giải các phương trình sau:
a)  $ 2\cos ^2 \dfrac{x}{2}+\sqrt{3}\sin x=1+2\sin 3x $
b) $ 3 \tan^2 x+4\tan x+4\cot x+3\cot^2 x+2=0 $

Bài 2. Gọi S là tập tất cả các số tự nhiên gồm 4 chữ số phân biệt được chọn từ các số 1;2;3;4;5;6;7. Lấy ngẫu nhiên một số từ S. Tính xác suất để lấy được số có mặt chữ số 6.

Bài 3. Trong khai triển của $ \left(2x^3-\dfrac{3}{x^2}\right)^n $ với $ n $ là số nguyên dương thỏa $ 2C_{n+6}^{5}=7A_{n+4}^3, $ tìm số hạng không chứa $ x? $

Bài 4. Tìm số hạng đầu và công sai của cấp số cộng $ (u_{n}) $ biết rằng công sai của $ (u_{n}) $ là số nguyên dương và
$u_{1}+u_{3}+u_{5}=15, \dfrac{1}{u_{1}}+\dfrac{1}{u_{3}}+\dfrac{1}{u_{5}}=\dfrac{59}{45} $.

Bài 5. Trong mặt phẳng tọa độ Oxy, cho điểm $ I(2;-5) $ và đường thẳng $ d:3x-2y+3=0. $ Viết phương trình đường thẳng $ d’ $ là ảnh của $ d $ qua phép đối xứng tâm $ I. $

Bài 6. Cho hình chóp $ S.ABCD $ có đáy $ ABCD $ là hình thang có $ AD $ là đáy lớn, $ AD=2BC. $ Gọi $ O $ là giao điểm của $ AC $ và $ BD. $ Gọi $ G_{1},G_{2} $ lần lượt là trọng tâm $ \Delta SCD, \Delta SAB, \ E $ là trung điểm $ SD. $
a)  Mặt phẳng $ (BCE) $ cắt $ SA $ tại $ F. $ Chứng minh: $ F $ là trung điểm $ SA. $
b) Chứng minh $ G_{1}G_{2} \parallel (SAD) $
c) Chứng minh $ (OG_{1}G_{2}) \parallel (SBC) $
d) Gọi $ M $ là điểm trên cạnh $ AB $ sao cho $ AB=4AM. $ Mặt phẳng $ (P) $ qua $ M $ và song song với $ BC, SD. $ Xác định thiết diện của hình chóp với mặt phẳng $ (P). $ Thiết diện là hình gì?

Hết

Đáp án

[userview]

Bài 1.

a) Phương trình tương đương với
$$
\begin{aligned}
& \cos x+\sqrt{3} \sin x=2 \sin 3 x \\
\Leftrightarrow & \frac{1}{2} \cos x+\frac{\sqrt{3}}{2} \sin x=\sin 3 x \\
\Leftrightarrow & \sin \left(x+\frac{\pi}{6}\right)=\sin 3 x \\
\Leftrightarrow x+\frac{\pi}{6}=3 x+k 2 \pi \text { hoặc } x+\frac{\pi}{6}=\pi-3 x+k 2 \pi \\
\Leftrightarrow x=\frac{\pi}{12}+k \pi \text { hoặc } x=\frac{5 \pi}{24}+\frac{k \pi}{2}, k \in \mathbb{Z}
\end{aligned}
$$

Bài 2. Gọi $\overline{a b c d}(a \neq 0)$ là số tự nhiên có 4 chữ số phân biệt được chọn từ các chữ số 1,2,3,4,5,6,7 .
$\overline{a b c d}:$ Có $A_{7}^{4}=840$ số tự nhiên có 4 chữ số phân biệt được chọn từ các chữ số 1,2,3,4,5,6,7
$\Longrightarrow|\Omega|=840$Gọi A là biên có sao cho số dược lậy là một số có mặt chữ số $6 .$
$$
|A|=4 . A_{6}^{3}=480 \Longrightarrow P(A)=\frac{|A|}{|\Omega|}=\frac{4}{7}
$$

Bài 3. 

\begin{aligned}
&2 C_{n+6}^{5}=7 A_{n+4}^{3} \Longleftrightarrow 2 \cdot \frac{(n+6) !}{5 !(n+1) !}=7 \cdot \frac{(n+4) !}{(n+1) !} \Longleftrightarrow \frac{(n+6) !}{(n+4) !}=420 \Longleftrightarrow(n+6)(n+5)=\\
&420 \Longleftrightarrow n^{2}+11 n-390=0 \Longleftrightarrow\left[\begin{array}{l}
n=15 \\
n=-26
\end{array} \Longleftrightarrow n=15(\text { vì n là số tự nhiên })\right.\\
&\text { Công thức } \mathrm{SHTQ}: T_{k+1}=C_{15}^{k} \cdot\left(2 x^{3}\right)^{15-k} \cdot\left(-\frac{3}{x^{2}}\right)^{k}=C_{15}^{k} \cdot 2^{15-k} \cdot(-3)^{k} \cdot x^{45-5 k}\\
&\text { Để số hạng không chứa } x \Longleftrightarrow 45-5 k=0 \Longleftrightarrow k=9 \text { . }\\
&\text { Vậy số hạng không chứa } \mathrm{x}: T_{10}=C_{15}^{9} .2^{6} \cdot(-3)^{9}=-6304858560 \text { . }
\end{aligned}

Bài 4. $\left\{\begin{array}{l}
u_{1}+u_{3}+u_{5}=15(1) \\
\frac{1}{u_{1}}+\frac{1}{u_{3}}+\frac{1}{u_{5}}=\frac{59}{45}(2) \end{array} \right.$
$(1) \Longleftrightarrow 3 u_{3}=15 \Longleftrightarrow u_{3}=5 $
$(2) \Longleftrightarrow \frac{1}{u_{1}}+\frac{1}{5}+\frac{1}{u_{5}}=\frac{59}{45} \Longleftrightarrow \frac{1}{u_{1}}+\frac{1}{u_{5}}=\frac{10}{9} $

$\Longleftrightarrow 9\left(u_{1}+u_{5}\right)=10 u_{1} u_{5} $

$\Longleftrightarrow 9.2 u_{3}= 10\left(u_{3}-2 d\right)\left(u_{3}+2 d\right)$

$\Longleftrightarrow 90=10\left(u_{3}^{2}-4 d^{2}\right)=25-4 d^{2}=9 $

$\Longleftrightarrow d^{2}=4$

$\Longleftrightarrow d=2(\text{vì} d>0) $
$u_{3}=5 \Longleftrightarrow u_{1}+2 d=5 \Longleftrightarrow u_{1}=5-2 d=1$.
và $u_{1}=1,d=2$

Bài 5. 

Gọi $M^{\prime}\left(x^{\prime} ; y^{\prime}\right)$ là ảnh của $\mathrm{M}$ qua phép đối xứng tâm $\mathrm{I} \Longleftrightarrow \mathrm{I}$ là trung điểm của $\mathrm{MM}^{\prime} \Longleftrightarrow$
$$
\left\{\begin{array}{l}
x_{I}=\frac{x_{M}+x_{M^{\prime}}}{2} \\
y_{I}=\frac{y_{M}+y_{M^{\prime}}}{2}
\end{array} \Longleftrightarrow\left\{\begin{array}{l}
4=x+x^{\prime} \\
-10=y+y^{\prime}
\end{array} \Longleftrightarrow\left\{\begin{array}{l}
x=4-x^{\prime} \\
y=-10-y^{\prime}
\end{array}\right.\right.\right.
$$
Ta có: $3 x-2 y+3=0 \Longleftrightarrow 3\left(4-x^{\prime}\right)-2\left(-10-y^{\prime}\right)+3=0 \Longleftrightarrow 12-3 x^{\prime}+20+2 y^{\prime}+3=0 \Longleftrightarrow$
$3 x^{\prime}-2 y^{\prime}-35=0$
Vậy M’ thuộc dường thẳng d’:3x-2y-35=0.
Vậy ảnh của đường thẳng d qua phép đối xứng tâm I là đường thẳng $\mathrm{d}^{\prime}: 3 \mathrm{x}-2 \mathrm{y}-35=0 .$

Bài 6. 

a) Ta có: $C \in(S A C) \cap(B C E)(1)$.
Trong $(S B D)$ gọi $\mathrm{K}$ là giao diểm của $\mathrm{SO}$ và $\mathrm{BE}$ mà $S O \subset(S A C), B E \subset(B C E)=K \in$
$(S A C) \cap(B C E)(2)$
$(1)(2) \Longrightarrow C K=(S A C) \cap(B C E)$
Trong $(S A C)$ gọi $\mathrm{F}$ là giao điểm của $\mathrm{SA}$ và $\mathrm{CK}$ mà $\mathrm{CK} \subset(B C E)=F=\operatorname{SAn}(B C E) .$ $\mathrm{Vi} A D \| B C=\frac{O C}{O A}=\frac{O B}{O D}=\frac{B C}{A D}=\frac{1}{2} \Longleftrightarrow \frac{C O}{C A}=\frac{B O}{B D}=\frac{1}{3}$
Xét $\triangle S O D$ : Áp dụng định lý Menelaus với 3 điểm $\mathrm{B}, \mathrm{K}, \mathrm{E}$ thẩng hàng ta có:
$\frac{C O}{C A} \cdot \frac{K S}{K O} \cdot \frac{F A}{F S}=1 \Longleftrightarrow \frac{F A}{F S}=1 \Longleftrightarrow \mathrm{F}$ là trung điẻm $\mathrm{SA}$
b) Trong (SAB), goi P là giao điểm của $S G_{1}$ và AB. Vì $G_{1}$ là trọng tâm của $\triangle S A B=P$
là trung điểm của AB.

Trong (SCD), gọi P là giao điểm của $S G_{2}$ và CD. Vì $G_{2}$ là trọng tàm của $\triangle S C D=\mathrm{Q}$
là trung điểm của CD. Xét $\triangle S P Q$ ta có: $\frac{S G_{1}}{S P}=\frac{2}{3}=\frac{S G_{2}}{S Q}=G_{1} G_{2} \| P Q(3)$

Xét hình thang ABCD ta có: PQ là đường trung bình của hình thang ABCD (do P,Q làn
lượt là trung điểm của $\mathrm{AB}, \mathrm{CD} \Longrightarrow P Q \| A D(4)$
$$
\text { Tì }(3)(4)=G_{1} G_{2}\left\|A D, \operatorname{mà} \mathrm{AD} \subset(\mathrm{SAD})=G_{1} G_{2}\right\|(S A D)
$$
c) Ta có: $G_{1} G_{2} \| A D$ mà $A D\left\|B C=G_{1} G_{2}\right\| B C=G_{1} G_{2} \|(S B C)(5)$
Trong (SAB), gọi H là giao điểm của $A G_{1}$ và $\mathrm{SB}$. Vì $G_{1}$ là trọng tần của $\triangle S A B=\mathrm{H}$
là trung điểm của $\mathrm{SB}$. Xét $\triangle H A C$ ta có: $\frac{A O}{A C}=\frac{2}{3}=\frac{A G_{1}}{A H}=O G_{1}\left\|C H \operatorname{mà} C H \subset(S B C)=O G_{1}\right\|(S B C)(6)$
Tì $(5)(6)=\left(O G_{1} G_{2}\right) \|(S B C)$
d) Ta có: $M \in(P) \cap(A B C D) \operatorname{mà}(P)\left\|B C=(P) \cap(A B C D)=x M x^{\prime}\right\| B C$.
Trong (ABCD), gọi N là giao diểm của xMx’ và CD.
Ta có: $N \in(P) \cap(S C D) \operatorname{mà}(P)\left\|S D=(P) \cap(S C D)=y N y^{\prime}\right\| S D$
Trong (SCD) gọi I là giao diểm của yNy’ và SC.
Ta có: $I \in(P) \cap(S B C) \operatorname{mà}(P)\left\|B C \Longrightarrow(P) \cap(S B C)=t I t^{\prime}\right\| B C .$
Trong (SBC), gọi J là giao điểm của tIt’ và SB. $((P) \cap(A B C D)=M N$
$\Longrightarrow$ thiệt diê
Ta có: $M N\|I J\| A D=M N I J$ là hình thang.

[/userview]

Đề thi cuối khóa STAR 2017 -2018: Toán 8

Đề bài

Bài 1. Giải các phương trình sau:

a) $ x^2 – 4x + 3 = 0$

b) $ \dfrac{1}{x-1} + \dfrac{2x^2 -5}{x^3 – 1} = \dfrac{4}{x^2 + x +1}$

c) $ |x-3| -3x = 1 $

d) $(x+3)^4 + (x+ 5)^4 = 2$

Bài 2. Giải các bất phương trình sau:

a) $ x – 5 > -5x + 3 $

b) $ \dfrac{2x-3}{-4 } \ge \dfrac{4-x}{-3}$

c) $ x^2 – 3x + 2 \le 0 $

d) $ \dfrac{x+1}{991} + \dfrac{x+5}{995} < \dfrac{x+4}{994} + \dfrac{x+9}{999}. $

Bài 3. 

a)  Quãng đường từ $ A $ đến $ B $ dài 180 $ km $. Xe thứ nhất khởi hành từ $ A $ đến $ B $. Cùng lúc đó và trên quãng đường $ AB $, xe thứ hai khởi hành từ $ B $ đến $ A $ với vận tốc lớn hơn vận tốc xe thứ nhất là $ 10km/h $. Biết hai xe gặp nhau tại nơi cách $ A $ là $ 80km/h $. Tính vận tốc của mỗi xe.

b) Dân số hiện nay của phường 12, quận 10 là 41618 người. Cách đây 2 năm dân số của phường là 40000 người. Hỏi trung bình mỗi năm dân số của phường đã tăng bao nhiêu phần trăm? ( giả sử \% tăng dân số mỗi năm là như nhau)

Bài 4. Một ngọn đèn đặt trên cao ở vị trí $A$, hình chiếu vuông góc của nó trên mặt đất là $H$. Người ta đặt 2 chiếc cọc có cùng độ cao là $1,6m$, thẳng đứng ở 2 vị trí $B$ và $C$ và 2 điểm $ B $, $ C $ thẳng hàng với $H$. Khi đó bóng cọc ở 2 vị trí $ B $, $ C $ ở trên mặt đất có độ dài lần lượt là $0,4m$ và $0,6m$. Biết $BC = 1,4m$. Hãy tính độ cao $AH$ của cột đèn.

Bài 5. Cho tam giác $ABC$ nhọn, các đường cao $ AD, BE, CF $ cắt nhau tại $ H $. Chứng minh rằng:
a) $ AF\cdot AB = AE\cdot AC $ và $ HF\cdot HC = HE\cdot HB. $
b) $ BE $ là phân giác của $ \widehat{DEF} $ . Từ đó chứng minh $ H $ là giao điểm các đường phân giác của $ \Delta DEF $.
c) $ BH\cdot BE + CH\cdot CF = BC^2 $
d)  Gọi $ O $ là giao điểm 3 đường trung trực, $ G $ là trọng tâm. Chứng minh $ G, H, O $ thẳng hàng và $ \dfrac{OG}{GH} = \dfrac{1}{2} $.

 

Đáp án Toán PTNK 2017

Bài 1. (Toán chung)  Tam giác $ABC$ nội tiếp đường tròn $(T)$ tâm $O$, bán kính $R$; $\angle BAC = 120^\circ $, $\angle ABC = 45^\circ $, $H$ là trực tâm. $AH$, $BH$, $CH$ lần lượt cắt $BC$, $CA$, $AB$ tại $M$, $N$, $P$.
a. Tính $AC$ theo $R$. Tính số đo góc $\angle HPN $ và $\dfrac{MP}{MN}$
b. Dựng đường kính $AD$, $HD$ cắt $(T)$ tại $E$ ($E \ne D$) và cắt $BC$ tại $F$. Chứng minh các điểm $A$, $N$, $H$, $P$, $E$ cùng thuộc một đường tròn và $F$ là trung điểm của $HD$.
c. Chứng minh $AD \bot NP$. Tia $OF$ cắt $(T)$ tại $I$, chứng minh $I$ là tâm đường tròn ngoại tiếp tam giác $HBC$ và $AI$ đi qua trung điểm của $MP$

Gợi ý

a.

  • Ta có $\angle AOC = 2 \angle ABC = 90^\circ$ (góc ở tâm bằng 2 lần góc nội tiếp cùng chắn 1 cung).
  • Suy ra tam giác $OAC$ vuông tại $O$, suy ra $AC^2 = OA^2 + OC^2 = 2R^2 \Rightarrow AC = R\sqrt{2}$. Tứ giác $BNPC$ có $\angle BNC = \angle BPC =90^\circ$ nên là tứ giác nội tiếp.
    Suy ra $\angle HPN = \angle HBC = 90^\circ – \angle ACB = 75^\circ$.
  • Các tứ giác $ANBM$ và $BNPC$ nội tiếp nên $\angle ANM = \angle ABC = 45^\circ, \angle CNP = \angle PBC = 45^\circ$.
  • Suy ra $\angle MNP = \angle CNP + \angle CPN = 90^\circ$.
    Và $\angle NPB = \angle ACB = \angle APM = 15^\circ$, suy ra $\angle NPM = \angle NPB + \angle APM = 30^\circ$.
  • Khi đó $\dfrac{MN}{MP} = \sin \angle NPM = \sin 30^\circ = \dfrac{1}{2}$. Suy ra $\dfrac{MP}{MN} = 2$.

b.

  • Ta có $\angle AEF = 90^\circ$ (góc nội tiếp chắn nửa đường tròn).
    Ta có $\angle ANH = \angle AEH = \angle APH = 90^\circ$ nên 5 điểm $A, N, H, P E$ cùng thuộc đường tròn đường kính $AH$.
  • Ta có $\angle ABD = \angle ACD = 90^\circ$ (góc nội tiếp chắn nửa đường tròn),
    suy ra $AB \bot BD$, suy ra $HC || BD$.
  • Tương tự ta có $HB \bot CN, \angle CD \bot CN$, suy ra $HB||CD$.
  • Tứ giác $HBDC$ có các cặp cạnh đối song song nên là hình bình hành, suy ra $F$ là trung điểm của $BC$ và $HD$.

c.

  • Ta có $\angle CAD = 45^\circ = \angle CNM$, suy ra $AD||MN$. Mà $MN \bot NP$, suy ra $AD \bot NP$.
  • Ta có $OF$ là trung trực của $BC$, suy ra $IB = IC$. $\angle BDC = 180^\circ – \angle BAC = 60^\circ$.
  • Xét tam giác $IOC$ có $\angle IOC = \dfrac{1}{2}\angle BOC = \angle 60^\circ$. Suy ra tam giác $IBC$ đều.
  • Do đó $IB =IC = IO$. (1)
  • Mặt khác tứ giác $HBOC$ có $\angle BHC + \angle BOC = 60^\circ + 120^\circ = 180^\circ$, suy ra $HBOC$ nội tiếp. (2)
  • Từ (1) và (2) suy ra $I$ là tâm đường tròn ngoại tiếp tam giác $HBC$.
  • Tam giác $PBC$ có $\angle BPC = 90^\circ, \angle PBC = 45^\circ$ nên là tam giác vuông cân,
    suy ra $PB = PC$, suy ra $P$ thuộc trugn trực của BC. Do đó $P, O, I$ thẳng hàng và $PI \bot BC$, suy ra $PI||AM$.
  • Mặt khác ta có $\angle BIH = 2\angle HCB = 90^\circ$, suy ra $HBMI$ nội tiếp, suy ra $\angle IMC = \angle IBH = 45^\circ$.
  • Suy ra $\angle IMC = \angle PBC = 45^\circ$, suy ra $IM||PA$.
  • Tứ giác $APIM$ có 2 cặp cạnh đối song song nên là hình bình hành, suy ra $AI$ qua trung điểm của $MP$.

Bài 2. (Toán chuyên) Cho tam giác $ABC$ nhọn nội tiếp đường tròn $(O)$ với tâm $O$. Gọi $D$ là điểm thay đổi trên cạnh $BC$ ($D$ khác $B,\,C$). Các đường tròn ngoại tiếp các tam giác $ABD$ và $ACD$ lần lượt cắt $AC$ và $AB$ tại $E$ và $F$ ($E$, $F$ khác $A$). Gọi $K$ là giao điểm của $BE$ và $CF$.
a. Chứng minh rằng tứ giác $AEKF$ nội tiếp.
b. Gọi $H$ là trực tâm tam $ABC$. Chứng minh rằng nếu $A,\,O,\,D$ thẳng hàng thì $HK$ song song với $BC$.
c. Ký hiệu $S$ là diện tích tam giác $KBC$. Chứng minh rằng khi $D$ thay đổi trên cạnh $BC$ ta luôn có $S\le \left(\dfrac{BC}{2}\right)^2 \tan \dfrac{\widehat{BAC}}{2}$.
d. Gọi $I$ là tâm đường tròn ngoại tiếp tam giác $AEF$. Chứng minh rằng $BF.BA-CE.CA=BD^2-CD^2$ và $ID$ vuông góc với $BC$.

Gợi ý

a.

  • Tứ giác $AEDB$ $\Rightarrow$ $\widehat{AEB}=\widehat{ADB}$, tứ giác $AFDC$ nội tiếp $\Rightarrow$ $\widehat{AFC}=\widehat{ADC}$.
  • Suy ra $\widehat{AEK}+\widehat{AFD}=\widehat{ADB}+\widehat{ADC}=180^o$.

b.

  • Ta có $\widehat{BKC}=\widehat{DKE}=180^o-\widehat{BAC}$ và $\widehat{BHC}=180^o-\widehat{BAC}$.
  • Suy ra $\widehat{BKC}=\widehat{BHC}$ $\Rightarrow$ $BHKC$ nội tiếp.
  • Suy ra $\widehat{FKH}=\widehat{HBC}=\widehat{HAC}$ và $\widehat{KCB}=\widehat{BAD}$.
  • Khi $A,\,O,\,D$ thẳng hàng, ta có $\widehat{BAD}=\widehat{BAO}=\widehat{HAC}$.
  • Do đó $\widehat{FKH}=\widehat{KCB}$ suy ra $KH//BC$

c.

  • Ta có $K$ thuộc cung $BHC$ của đường tròn ngoại tiếp tam giác $BHC$ tâm $T$.
  • Gọi $M$ là trung điểm của $BC$ và $N$ là điểm chính giữa cung $BHC$.
  • Dựng $KL\perp BC$, ta có $KL\le TN-TM=MN$.
  • Mà $\dfrac{MN}{BC}=\tan \dfrac{\widehat{NBM}}{2}=\tan \dfrac{\widehat{BAC}}{2}$, suy ra $MN=\tan \dfrac{\widehat{BAC}}{2}.\dfrac{BC}{2}$.
  • Do đó $S_{BKC}=\dfrac{1}{2}.KL.BC\le \dfrac{BC^2}{4}\tan \dfrac{\widehat{BAC}}{2}$.

d.

  • Xét tam giác $BCF$ và tam giác $BDA$ có $\widehat{BCF}=\widehat{BAD}$ và góc $B$ chung.
  • Suy ra $\Delta BFC\sim \Delta BDA$ $\Rightarrow$ $\dfrac{BD}{BA}=\dfrac{BF}{BC}$ $\Rightarrow$ $BF.BA=BD.BC$.
  • Chứng minh tương tự ta có $CE.CA=CB.CD$.
  • Suy ra $BF.BA-CE.CA=BC.BD-BC.CD=BC(BD-CD)=(BD+BC)(BD-BC)=BD^2-CD^2$.
  • Ta có $\widehat{ADF}=\widehat{ACF}=\widehat{AEB}-\widehat{EKC}=\widehat{AEB}-\widehat{A}$
  • và $\widehat{ADE}=\widehat{ABE}=\widehat{AFC}-\widehat{A}$,suy ra $\widehat{EDF}=\widehat{ADF}+\widehat{ADE}=\widehat{AEB}+\widehat{AFC}-2\widehat{A}=180^o-2\widehat{A}=\widehat{EIF}$.
  • Do đó tứ giác $IEDF$ nội tiếp, hơn nữa $IE=IF$ nên $DI$ là phân giác $\widehat{EDF}$.
  • Mặt khác $\widehat{FDB}=\widehat{BAC}=\widehat{CDE}$.
  • Suy ra $DB,\,DI$ lần lượt là phân giác ngoài và phân giác trong của $\widehat{EDF}$. Vậy ta có điều phải chứng minh.